You are on page 1of 229

UNIT 1 A

Basic mathematics for management

Unit 1
BBM 511

Quantitative Techniques

Basic Mathematics for Management

B WAWASAN OPEN UNIVERSITY


BBM 511 Quantitative Techniques

COURSE TEAM
Course Team Coordinator: Mr. Yeoh Hong Beng Course Coordinator: Ms. Lum Li Sean Content Writers: Professor M. P. Gupta, Dr. Arun Kanda, Dr. Ashish Chatterjee, Dr. J. K. Sharma, Professor P. K. Bhowmik, Professor Rakesh Khurana, Dr. B. B. Khanna and Professor G. Sambasiva Rao Course Team Members: Professor Dr. Ng Wai Kong and Mr. Lai Choo Heng

PRODUCTION
In-house Editor: Ms. Jeanne Chow Graphic Designers: Ms. Patsy Yap and Chrisvie Ong

Wawasan Open University is Malaysias first private not-for-profit tertiary institution dedicated to adult learners. It is funded by the Wawasan Education Foundation, a tax exempt entity established by the Parti Gerakan Rakyat Malaysia and supported by the Yeap Chor Ee Charitable and Endowment Trusts, other charities, corporations and members of the public.

Indira Gandhi National Open University, 1988 All rights reserved. No part of this work may be reproduced in any form, by mimeograph or any other means, without permission in writing from Indira Gandhi National Open University. Further information on the Indira Gandhi National Open University courses may be obtained from the Universitys Office at Maidan Garni, New Delhi 110 068.

UNIT 1 C
Basic mathematics for management

Contents
Unit 1 Basic Mathematics for Management
Overview Objectives 1 Quantitative decision making
Objectives Introduction 1.1 Meaning of quantitative techniques 1.2 Statistics and operations research 1.3 Classification of statistical methods 1.4 Models in operations research (OR) 1.5 Various statistical techniques 1.6 Advantages of quantitative approach to management 1.7 Quantitative techniques in business and management 1.8 Use of computers Keywords Suggested answers to self-assessment exercises 3 3

5 5 5 7 7 11 12 17 19 20 22 23 26

Functions and progressions


Objectives Introduction 2.1 Definitions 2.2 Types of function 2.3 Solution of functions

31 31 31 31 35 43

D WAWASAN OPEN UNIVERSITY


BBM 511 Quantitative Techniques

2.4 Business applications 2.5 Sequence and series 2.6 Arithmetic progression (AP) 2.7 Geometric progression (GP) Keywords Suggested answers to activities

45 47 48 50 55 57

Basic calculus and its applications


Objectives Introduction 3.1 Limit and continuity 3.2 Concept of slope and rate of change 3.3 Concept of derivative 3.4 Rules of differentiation 3.5 Applications of the derivative 3.6 Concept of maxima and minima with managerial applications Keywords Suggested answers to activities

65 65 65 66 73 77 79 86 94

100 102

Matrix algebra and its applications


Objectives Introduction 4.1 Matrices: Definition and notations 4.2 Some special matrices 4.3 Matrix representation of data 4.4 Operations on matrices

109 109 109 110 111 112 115

UNIT 1 E
Basic mathematics for management

4.5 Determinant of a square matrix 4.6 Inverse of a matrix 4.7 Solution of linear simultaneous equations 4.8 Applications of matrices Keywords Suggested answers to activities

122 126 131 140 146 148

Summary of Unit 1

155

F WAWASAN OPEN UNIVERSITY


BBM 511 Quantitative Techniques

UNIT 1 1
Basic mathematics for management

Course Overview

his course addresses quantitative techniques used in supporting managerial decision making. The course begins with an overview of quantitative models and basic mathematics for management. Subsequently, the course highlights data collection and presentation methods as well as measures of central tendency, variation and skewness which are used to summarise raw data collected from a survey. The course introduces concepts and applications of probability, discrete and continuous probability distributions and analysing decisions using marginal analysis and decision tree. Sampling distributions that can be formed to relate sample information with the unknown population parameters are also explained in this course. This course also focuses on various hypothesis tests involving population means and proportions, goodness-of-fit as well as independence tests between two categorical variables. Finally, we end the course with business forecasting techniques including correlation, regression and time series models.

2 WAWASAN OPEN UNIVERSITY


BBM 511 Quantitative Techniques

UNIT 1 3
Basic mathematics for management

Overview
nit 1 aims to expose you to fundamental mathematics which is essential to comprehend issues in business and management that require the applications of mathematics. This unit consists of four sections. Section 1 provides an overview on quantitative techniques and models as well as statistical methods needed to support managerial decision making. This will lead you to other sections that have applications of these statistical methods. In section 2, you will be exposed to functional relationships between a dependent variable and a set of independent variables. Besides, the arithmetic and geometric progressions have also been defined. You will also learn to apply functions and progressions in the business-related context. Section 3 starts with concepts of limits, continuity and calculus. The concepts and applications of differential calculus are the main emphasis in this section. In the last section of the unit, you will learn matrix algebra and its applications in solving business-related problems.

Objectives
By the end of this unit, you should be able to: 1. Identify and describe various quantitative models, statistical methods and techniques that are frequently used in business decision making. 2. Define and draw graphs to show functional relationships between a set of quantitative variables. 3. Define arithmetic and geometric progressions. 4. Define limits, continuity of a function and slope of a tangent line of a function. 5. Define differential calculus. 6. Define matrix and its operations. 7. Apply the functions, progressions, calculus and matrix operations in business related context.

4 WAWASAN OPEN UNIVERSITY


BBM 511 Quantitative Techniques

UNIT 1 5
Basic mathematics for management

1 Quantitative Decision Making


Objectives
By the end of this section, you should be able to: 1. Explain the complexity of todays managerial decisions and define the meaning of quantitative techniques. 2. State the need of using quantitative approach to managerial decisions. 3. Explain the role of statistical methods in data analysis. 4. Describe various models frequently used in operations research and the basis of their classification. 5. Describe various statistical methods applied in business decision making. 6. Explain the areas of applications of quantitative approach in business and management.

Introduction
You may be aware of the fact that prior to the industrial revolution, individual business was small and production was carried out on a small scale basis to cater to local needs. The management of such business enterprises was very different from the present management of large scale business. The information needed by the decision maker (usually the owner) to make effective decisions was less extensive. Thus, owners make decisions based upon his past experience and intuition only. Some of the reasons for this were: 1. Marketing of products was not a problem because the owner knows the customers personally. There was hardly any competition in the business. 2. Test marketing of the product was not needed because the owner is familiar with the choices and requirements of the customers through personal interaction. 3. The manager (also the owner) also works with his workers at the shop floor. He knows all of them personally as the number of workers was small. This reduces the need to keep personal data. 4. The work was being carried out at the work centre itself. Thus, production records were not needed. 5. The owner could gather the information that he required through observation.

6 WAWASAN OPEN UNIVERSITY


BBM 511 Quantitative Techniques

Now, in the face of increasing complexity in business and industry, intuition alone has no place in decision making because basing a decision on intuition becomes highly questionable when the decision involves the choice among several courses of action; and each of these courses of action can achieve several management objectives simultaneously. Hence, there is a need to train people to manage a system both efficiently and creatively. Quantitative techniques have made valuable contributions towards arriving at effective decisions in various functional areas of management-marketing, finance, production and personnel. Today, these techniques are also widely used in regional planning, transportation, public health, communication, military, agriculture, etc. Quantitative techniques are being used extensively to aid business decision making, due to the following reasons: 1. Complexity of todays managerial activities which involve constant analysis of existing situations, setting objectives, seeking alternatives, implementing, coordinating, controlling and evaluating the decisions made. 2. Availability of different types of tools for quantitative analysis of complex managerial problems. 3. Availability of high speed computers to apply quantitative techniques (or models) to real life problems in all types of organisations such as business, industry, military, health, and so on. Computers have played an important role in arriving at the optimal solution of complex managerial problems both in terms of time and cost.

In spite of these reasons, the quantitative approach, however, does not totally eliminate the scope of qualitative or judgement ability of the decision maker. Of course, these techniques complement the experience and knowledge of decision maker in decision making.

UNIT 1 7
Basic mathematics for management

1.1 Meaning of quantitative techniques


Quantitative techniques refer to the group of statistical, and operations research (or programming) techniques as shown in the following chart. All these techniques require preliminary knowledge of certain topics in mathematics as discussed in section 2 later.

Quantitative techniques Statistical techniques Operations research (or programming) techniques

The quantitative approach in decision making requires that, problems be defined, analysed and solved in a conscious, rational, systematic and scientific manner based on data, facts, information, and logic and not on mere whims and guesses. In other words, quantitative techniques (tools or methods) provide the decision maker a scientific method based on quantitative data in identifying a course of action among the given list of courses of action to achieve the optimal value of the predetermined objective or goal. One common characteristic of all types of quantitative techniques is that numbers, symbols or mathematical formulae (or expressions) are used to represent the models of reality.

1.2 Statistics and operations research


Statistics
The word statistics can be used in a number of ways. Commonly it is described in two senses namely: 1. Plural sense (Statistical data) The plural sense of statistics means some sort of statistical data. When it means statistical data, it refers to the numerical description of quantitative aspects of things. These descriptions may take the form of counts or measurements. For example, statistics of students of a college include the number of students, the number of male and females, married and unmarried, or undergraduates and postgraduates. They may also include measurements such as heights and weights. 2. Singular sense (Statistical methods) The large volume of numerical information (or data) gives rise to the need for systematic methods which can be used to collect, optimise or classify, present, analyse and interpret the information effectively for the purpose of making wise decisions. Statistical methods include all those devices of analysis and synthesis by means of which statistical data are systematically collected and used to explain or describe a given phenomena.

8 WAWASAN OPEN UNIVERSITY


BBM 511 Quantitative Techniques

The above mentioned five functions of statistical methods are also called phases of a statistical investigation. A major part of Unit 2 (section 5 to section 8) is devoted to the methods used to analyse the presented data. There are various methods used to analyse data that contain a range of simple to sophisticated mathematical techniques. However, in Units 2 to 5 of this course, only the most commonly used methods of statistical analysis are included. As an illustration, suppose that we are interested in knowing the income level of the people living in KL. For this we may adopt the following procedures: 1. Data collection: The following data is required for this purpose: a. Population of the city. b. Number of individuals who are getting income. c. Daily income of each working individual.

2. Organising the data: The data so obtained should now be organised in different income groups. This will reduce the bulk of the data.

3. Presentation: The organised data may now be presented by means of various types of graphs or other visual aids. Data presented in an orderly manner facilitates statistical analysis.

4. Analysis: On the basis of systematic presentation (tabular form or graphical form), determine the average income of an individual and the extent of disparities that exist. This information will help to get an understanding of the phenomenon (i.e., income of individuals).

5. Interpretation: All the above steps may now lead to drawing conclusions which will aid in decision making a policy decision for improvement of the existing situation.

UNIT 1 9
Basic mathematics for management

Characteristics of data
It is probably more common to refer to data in quantitative form as statistical data. But not all numerical data is statistical. In order for numerical descriptions to be called statistics, they must possess the following characteristics: 1. They must be an aggregate of facts. Single unconnected figures cannot be used to study the characteristics of the phenomenon. For instance, in the market survey of a new product, demographic factors and buying behaviours of potential customers are collected to predict the demand of the product. 2. They should be affected by a multiplicity of causes to a marked extent. For example, in social services, the observations recorded are affected by a number of factors (controllable and uncontrollable) such as the direct response from those involved and the impact from society in the social services. 3. They must be enumerated or estimated according to reasonable standards of accuracy. For example, in the measurement of height, one may measure correct up to 0.01 of a cm; the quality of the product is estimated by certain tests on small samples drawn from a big lot of products. 4. They must be collected in a systematic manner for a pre-determined purpose. Facts collected in a haphazard manner and without a complete awareness of the object, will be confusing and cannot be made the basis of valid conclusions. For example, the collected data on price serves no purpose unless one knows whether he wants to collect data on wholesale or retail prices and what are the relevant commodities in view. 5. They must be placed in relation to each other. That is, data collected should be comparable, otherwise these cannot be placed in relation to each other, e.g., statistics on the yield of crop and quality of soil are related but these yields cannot have any relation with the statistics on the health of the people. 6. They must be numerically expressed. That is, any facts to be called statistics must be numerically or quantitatively expressed. Qualitative characteristics such as beauty, intelligence, etc., cannot be included in statistics unless they are quantified.

10 WAWASAN OPEN UNIVERSITY


BBM 511 Quantitative Techniques

Types of statistical data


An effective managerial decision concerning a problem on hand depends on the availability and reliability of statistical data. Statistical data can be broadly grouped into two categories: 1. Secondary (or published) data. 2. Primary (or unpublished) data.

The secondary data are those which have already been collected by another organisation and are available in the published form. You must first check whether any such data is available on the subject matter of interest and make use of it, since it will save considerable time and money. But the data must be scrutinised properly since it was originally collected, perhaps for another purpose. The data must also be checked for reliability, relevance and accuracy. A great deal of data is regularly collected and disseminated by international bodies such as World Bank, Asian Development Bank, International Labour Organisation, Secretariat of United Nations, etc., and also Malaysian government and its many agencies such as Bank Negara Malaysia, Statistics Department, Ministry of International Trade and Industry (MITI), Private Research Organisations, Trade Associations, etc. When secondary data is not available or not reliable, you would need to collect original data to suit your objectives. Original data collected specifically for a current research are known as primary data. Primary data can be collected from customers, retailers, distributors, manufacturers or other information sources. Primary data may be collected through any of the three methods: observation, survey, and experimentation. Data are also classified as micro and macro. Micro data relate to a particular unit or region whereas macro data relate to the entire industry, region or economy.

Operations Research
You would recall that in Operations Research, a mathematical model to represent the situation under study is constructed. This helps in two ways, either to predict the performance of the system under certain controls or to determine the action/control needed to optimise performance.

UNIT 1 11
Basic mathematics for management

1.3 Classification of statistical methods


By now, you may have realised that effective decisions have to be based upon realistic data. The field of statistics provides the methods for collecting, presenting and meaningfully interpreting the given data. Statistical methods broadly fall into three categories as shown in the following chart.

Statistical methods Descriptive statistics Data collection Data presentation Inductive statistics Statistical inference Estimation Statistical decision theory Analysis of business decision

Descriptive statistics
There are statistical methods which are used for re-arranging, grouping and summarising sets of data to obtain better information of facts and thereby better description of the situation that can be made. For example, changes in the consumer price index, yield by wheat, etc., are frequently illustrated using different types of charts and graphs. These devices summarise large quantity of numerical data for easy understanding. Various types of averages, can also reduce a large mass of data to a single descriptive number. The descriptive statistics include the methods of collection and presentation of data, measure of central tendency and dispersion, trends, index numbers, etc.

Inductive statistics
It is concerned with the development of some criteria which can be used to derive information about the nature or characteristics of the members of entire groups (also called population or universe) from the nature or characteristics of the small portion (also called sample) of the given group. The specific values of the population members are called parameters and that of samples are called statistics. Thus, inductive statistics is concerned with estimating population parameters from the sample statistics and deriving a statistical inference. Samples are drawn instead of a complete enumeration for the following reasons: 1. The number of units in the population may not be known. 2. The population units may be too many in number and/or widely dispersed. Thus, complete enumeration is extremely time consuming and at the end of a full enumeration, so much time is lost that the data becomes obsolete by that time.

12 WAWASAN OPEN UNIVERSITY


BBM 511 Quantitative Techniques

3. It may be too expensive to include each population item.

Inductive statistics include the methods such as probability and probability distributions, sampling and sampling distributions, various methods of testing hypothesis, correlation, regression, factor analysis and time series analysis.

Statistical decision theory


Statistical decision theory deals with analysing complex business problems with alternative courses of action (or strategies) and possible consequences. Basically, it is to provide more concrete information concerning these consequences, so that the best course of action can be identified from alternative courses of action. Statistical decision theory relies heavily not only upon the nature of the problem on hand, but also upon the decision environment. Basically there are four different states of decision environment as given below: State of decision Certainty Risk Uncertainty Conflict Consequences Deterministic Probabilistic Unknown Influenced by an opponent

Since statistical decision theory also uses probabilities (subjective or prior) in analysis, therefore it is also called a subjectivist approach. It is also known as Bayesian approach because Bayes theorem is used to revise prior probabilities in the light of additional information.

1.4 Models in operations research (OR)


In this section, we are presenting several classifications of OR models so that you know more about the role of models in decision making:

Purpose
A model is the representation of a system which, in turn, represents a specific part of reality (an object of interest or subject of inquiry in real life). The means, representing a system may be physical, graphic, schematic, analogy, mathematical, symbolic or a combination of these. Through all these means, an attempt is made to abstract the essence of reality, which in turn, is quite helpful to describe, explain and predict the behaviour of the system. Thus, depending upon the purpose and the stage at which the model is developed, models can be classified into four categories.

UNIT 1 13
Basic mathematics for management

1. Descriptive model. Such model is used to describe the behaviour of a system based on certain information. For example, a model can be built to describe the behaviour of demand for an inventory item for a stated period, by keeping the record of various demand levels and their respective frequencies. A descriptive model is used to display the problem situation more vividly including the alternative choices to enable the decision maker to evaluate the results of each alternative choice. However, such model does not select the best alternative. 2. Explanatory model. Such model is used to explain the behaviour of a system by establishing relationships between its various components. For example, a model can be built to explain variations in productivity by establishing relationships among factors such as wages, promotion policy, education levels, etc. 3. Predictive model. Such model is used to predict the future status of a system based on data. For example, a model can be built to predict stock prices (within an industry group), for any given level of earnings per share. 4. Prescriptive (or normative) model. A prescriptive model is one which provides the norms for the comparison of alternative solutions which result in the selection of the best alternative (the most preferred course of action). Examples of such models are allocation models.

Degree of abstraction
The following chart shows the classification of models according to the degree of abstraction: Model Physical Graphic Schematic Analog Mathematical Most abstract Degree of abstraction Least abstract

Any three-dimensional model that looks like the real thing but is either reduced in size or scaled up, is a physical (conic) model. These models include city planning maps, plant layout charts, plastic models of airplanes, body parts, etc. These models are easy to observe, build and describe, but cannot be manipulated and used for prediction. An organisation chart showing responsibility relationships is an example of graphic model. A flow chart (or diagram) depicting the sequence of activities during the complete processing of a product is an example of schematic model. Another example

14 WAWASAN OPEN UNIVERSITY


BBM 511 Quantitative Techniques

of schematic model is the computer program where main features of the program are represented by a schematic description of steps. Analog models are closely associated with iconic models. However, they are not replicas of problem situations. Rather they are small physical systems that have similar characteristics and work like an object or system it represents, for example, childrens toys, model rail-roads, etc. These models might not allow direct handling or manipulation. Mathematical (or symbolic) models represent the systems (or reality) by using mathematical symbols and relationships. These are very precise and abstract, and can be manipulated by using laws of mathematics. The input-output model of national economy involving several objectives, constraints, inputs and inter-linkages between them is an example of representing a complex system with the help of a set of equations.

Degree of certainty
Models can also be classified according to the degree of assumed certainty. Under this classification, models are divided into deterministic versus probabilistic models. Models in which the selection of each course of action (or strategy) results in unique and known pay-off or consequence are called deterministic models. Examples of such models are linear programming, transportation and assignment models. Situations in which each course of action (or strategy) can result in more than one pay-off or consequence are called probabilistic models. Since the concept of probability is used in such models, therefore the pay-offs or consequences due to a managerial action cannot be predicted with certainty. Examples of such models are simulation models, decision theory models, etc.

Specified behaviour characteristics


The following chart describes the classification of models based on specified behaviour characteristics. Such type of classification helps in understanding the nature and role of models in representing the management and economic status of organisations.

UNIT 1 15
Basic mathematics for management

Classification according to behaviour characteristics Models Abstract Dynamic Non-linear Unstable (Constrained) Stable Linear Static Non-linear Stable Physical Dynamic Linear Unstable Stable Static

Unstable (explosive)

Transient (non-existent) Transient Steady state

Steady state

Realistic representation of corporate and economic behaviour (Source: Loomba, M P (1978) Management A Quantitative Perspective, New York: Macmillan Publishing Co.)

The models that are concerned with a particular set of fixed conditions and do not change in a short-term period (or planning period) are known as static models. This implies that such models are independent of time and only one decision is required for a given time period. For example, the resources required for a product and the technology or manufacturing process do not change in short-term period. Linear programming is the particular example of static models. On the other hand, there are certain types of problems where time factor plays an important role and there is an impact of changes over a period of time. In all such situations, the decision maker has to make a sequence of optimal decisions at every decision point (i.e., variable time) regardless of what the prior decision has been. The problem of product development in which the decision maker has to make decisions at every decision point such as product design, test-market, full-scale production, etc., is an example of dynamic model. Dynamic programming is the particular example of dynamic model. Linear models are those in which each component exhibits a linear behaviour. The word `linear is used to describe the relationship among two or more variables which are directly proportional. For example, if our resources increase by some percentage,

16 WAWASAN OPEN UNIVERSITY


BBM 511 Quantitative Techniques

then it would increase the output by the same percentage. If one or more components of a model exhibit a non-linear behaviour, then such models are called non-linear models. A mathematical model of the form Z = 5x + 3 is called a linear model whereas a model of the form Z = 5x2 + 3xy + y2 is called a non-linear model.

Procedure (or method) of solution


The type of procedure used to derive solutions to mathematical models can be classified into two categories: (1) analytical models, and (2) simulation models. An analytical model consists of a mathematical structure and is solved by known mathematical or analytical techniques to yield a general solution. Examples of analytical models are network models (PERT/CPM), linear programming models, game theory models, inventory control models. A simulation model is the experimentation (Computer assisted or manual) on a mathematical structure of real-life system. It is done by inserting specific values of decision variables into the given system under certain assumptions in order to describe and evaluate the system behaviour over a period of time. For example, we can test the effects of different number of service counters assuming different arrival rates of customers with respect to the total cost of providing service to customers. The following table summarises our discussion on classification of models. Criterion classification Purpose Degree of abstraction Degree of certainty Specified behaviour characteristics Procedure of solution Categories of OR models Descriptive, explanatory, predictive, prescriptive Physical, graphic, schematic, analog, mathematical Deterministic, probabilistic certainty, risk, uncertainty Static, dynamic, linear, non-linear Analytical, simulation

You have read about certain standard techniques or prototype models of operations research which can be helpful to a decision maker in solving a variety of problems.

UNIT 1 17
Basic mathematics for management

1.5 Various statistical techniques


A brief comment on certain standard techniques of statistics which can be helpful to a decision maker in solving problems is given below. However, each one of these techniques requires detailed studies and in our context we are merely listing these to arouse your interest. Measures of central tendency: Obviously for proper understanding of quantitative data, they should be classified and converted into a frequency distribution (number of times or frequency with which a particular data occurs in the given mass of data). This type of condensation of data reduces their bulk and gives a clear picture of their structure. If you want to know any specific characteristics of the given data or the frequency distribution of one set of data to be compared with another, then it is necessary that the frequency distribution itself must be summarised and condensed in such a manner that it must help us to make useful inferences about the data, and also to provide yardstick for comparing different sets of data. Measures of average or central tendency provide one such yardstick. Different methods of measuring central tendency provide us with different kinds of averages. The three main types of averages commonly used are: 1. Mean: The mean is the common arithmetic average. It is computed by dividing the sum of the values of the observations by the number of items observed. 2. Median: The median is that item which lies exactly half-way between the lowest and the highest value when the data is arranged in an ascending or descending order. It is not affected by the value of the observation but by the number of observations. Suppose you have the data on monthly income of households in a particular area. The median value would give you that monthly income which divides the number of households into two equal parts. Fifty per cent of all the households have a monthly income above the median value and fifty per cent of households have a monthly income below the median income. 3. Mode: The mode is the central value (or item) that occurs most frequently. When the data is organised as a frequency distribution, the mode is the category which has the maximum number of observations. For example, a shopkeeper ordering fresh stock of shoes for the season would make use of the mode to determine the shoe size which is most frequently sold. The advantages of mode are that (a) it is easy to compute (b) it is not affected by extreme values in the frequency distribution, and (c) it is representative if the observations are clustered at one particular value or class.

Measures of dispersion: The measures of central tendency measure the most typical value around which most values in the distribution tend to converge. However, there are always extreme values in each distribution. These extreme values indicate the spread or the dispersion of the distribution. The measures of this spread are called `measures of dispersion or `variation or `spread. Measures of dispersion would tell you the number of values which are substantially different from the mean, median

18 WAWASAN OPEN UNIVERSITY


BBM 511 Quantitative Techniques

or mode. The commonly used measures of dispersion are range, mean deviation and standard deviation. The data may spread around the central tendency in a symmetrical or an asymmetrical pattern. The measures of the direction and degree of symmetry are called measures of the skewness. Another characteristic of the frequency distribution is the shape of the peak, when it is plotted on a graph paper. The measures of the peakedness are called measures of Kurtosis. Correlation: Correlation coefficient measures the degree to which the change in one variable (the dependent variable) is associated with change in the other variable (independent one). For example, as a marketing manager, you would like to know if there is any relation between the amount of money you spend on advertising and the sales you achieve. Here, sales is the dependent variable and advertising budget is the independent variable. Correlation coefficient, in this case, would tell you the extent of relationship between these two variables, whether the relationship is directly proportional (i.e., increase or decrease in advertising is associated with increase or decrease in sales) or it is an inverse relationship (i.e., increasing advertising is associated with decrease in sales and vice-versa) or there is no relationship between the two variables. However, it is important to note that correlation coefficient does not indicate a casual relationship. Sales is not a direct result of advertising alone, there are many other factors which affect sales. Correlation only indicates that there is some kind of association; whether the relationship is casual or causal can be determined only after further investigation. You may find a correlation between the height of your salesmen and the sales, but obviously it is of no significance. Regression analysis: For determining causal relationship between two variables, you may use regression analysis. Using this technique you can predict the dependent variables on the basis of the independent variables. The correlation and regression analysis are suitable techniques to find the relationship between two variables only. But in reality, you would rarely find a one-to-one causal relationship; rather you would find that the dependent variables are affected by a number of independent variables. For example, sales can be affected by advertising budget, the media plan, the content of the advertisements, number of salesmen, price of the product, efficiency of the distribution network and a host of other variables. For determining causal relationship involving two or more variables, multivariate statistical techniques are applicable. The most important of these are the multiple regression analysis, discriminant analysis and factor analysis. Time series analysis: A time series consists of a set of data (arranged in some desired manner) recorded either at successive points in time or over successive periods of time. The changes in such type of data from time to time are considered as the resultant of the combined impact of a force that is constantly at work. This force has four components: (a) Time series data (b) Secular trend (c) Periodic changes, cyclical changes and seasonal variations, and (d) Irregular or random variations. With time series analysis, you can isolate and measure the separate effects of these forces on the variables. Examples of these changes can be seen, if you start measuring the increase in the cost of living, increase of population over a period of time, growth of agricultural food production in Malaysia over the last fifteen years, seasonal requirement of items, impact of floods and so on.

UNIT 1 19
Basic mathematics for management

Index numbers: Index number is a relative number that is used to represent the net result of change in a group of related variables over a period of time. Index numbers are stated in the form of percentages. For example, if we say that the index of prices is 105, it means that prices have gone up by 5% as compared to a point of reference, called the base year. If the prices of the year 2006 are compared with those of 2002, the year 2006 would be called given or current year and the year 2002 would be termed as the base year. Index numbers are also used in comparing production, sales price, volume employment, etc., to reflect the changes over a period of time relative to a base. Sampling and statistical inference: In many cases, due to shortage of time, cost or non-availability of data, only limited part or section of the universe (or population) is examined to (i) get information about the universe as clearly and precisely as possible, and (ii) determine the reliability of the estimates. This small part or section selected from the universe is called the sample, and the process of selecting the section (or part) is called sampling. The scheme of drawing samples from the population can be classified into two broad categories: 1. Random sampling schemes: In these schemes, drawing of elements from the population is random and the selection of an element is made in such a way that every element has equal chance (probability) of being selected. 2. Non-random sampling schemes: In these schemes, drawing of elements from the population is based on the choice or purpose of the selector.

The sampling analysis through the use of various `tests namely Z-normal distribution, students `t distribution; F-distribution and 2-distribution makes it possible to derive inferences about population parameters with specified level of significance and the given degrees of freedom. You will read about a number of tests in section 15 and 16 of this study material to derive inferences about population parameters.

1.6 Advantages of quantitative approach to management


Executives at all levels in business and industry come across the problem of making decision at every stage in their day-to-day activities. Quantitative techniques provide executives with scientific basis for decision making and enhance his/her ability to make long-range plans and to solve the daily problems of running a business and industry with greater efficiency and confidence. Let us now also look at some of the advantages of the study of statistics: 1. Definiteness: The study of statistics helps us in presenting general statements in a precise and a definite form. Statements of facts conveyed numerically are more precise and convincing than those stated qualitatively. For example, the statement that literacy rate as per 1981 census was 36% compared to 29% for 1971 census is more convincing than stating simply that literacy in our country has increased.

20 WAWASAN OPEN UNIVERSITY


BBM 511 Quantitative Techniques

2. Condensation: The new data is often unwieldy and complex. The purpose of statistical methods is to simplify large mass of data and to present meaningful information from them. For example, it is difficult to form a precise idea about the income position of the people of India from the data of individual income in the country. The data will be easy to understand and more precise if it can be expressed in the form of per capita income. 3. Comparison: According to Bodding, the object of statistics is to enable comparisons between past and present results with a view to ascertaining the reasons for change which have taken place and the effect of such changes in the future. Thus, if one wants to appreciate the significance of figures, then he must compare them with others of the same kind. For example, the statement per capita income has increased considerably shall not be meaningful unless some comparisons of past figures are made. This will help in drawing conclusions as to whether the standard of living of the people in Malaysia is improving. 4. Formulation of policies: Statistics provide the basic material for framing policies not only in business but also in other fields. For example, data on birth and mortality rate not only help in assessing future growth in population but also provide necessary data for framing a scheme of family planning. 5. Formulating and testing hypothesis: Statistical methods are useful in formulating and testing hypothesis (assumption) or statement and to develop new theories. For example, the hypothesis: whether a student has benefited from a particular media of instruction, can be tested by using appropriate statistical method. 6. Prediction: For framing suitable policies or plans, and then for implementation, it is necessary to have the knowledge of future trends. Statistical methods are highly useful for forecasting future events. For example, for a businessman to decide how many units of an item should be produced in the current year, it is necessary for him to analyse the sales data of the previous years.

1.7 Quantitative techniques in business and management


Some of the areas where statistics can be used are as follows:

Management
1. Marketing. a. Analysis of marketing research information. b. Statistical records for building and maintaining an extensive market.

UNIT 1 21
Basic mathematics for management

c. Sales forecasting.

2. Production. a. Production planning, control and analysis. b. Evaluation of machine performance. c. Quality control requirements. d. Inventory control measures.

3. Finance, accounting and investment. a. Financial forecast and budget preparation. b. Financial investment decisions. c. Selection of securities. d. Auditing function. e. Credit policies, credit risk and delinquent accounts.

4. Personnel. a. Labour turnover rate. b. Employment trends. c. Performance appraisal. d. Wage rates and incentive plans.

Economics
1. Measurement of gross national product and input output analysis. 2. Determination of business cycle, long-term growth and seasonal fluctuations. 3. Comparison of market prices, costs and profits of individual firms. 4. Analysis of population, land economics and economic geography. 5. Operational studies of public utilities.

22 WAWASAN OPEN UNIVERSITY


BBM 511 Quantitative Techniques

6. Formulation of appropriate economic policies and evaluation of their effects.

Research and development


1. Development of new product lines. 2. Optimal use of resources. 3. Evaluation of existing products.

Natural science
1. Diagnosing the disease based on data like temperature, pulse rate, blood pressure, etc. 2. Judging the efficacy of a particular drug for curing a certain disease. 3. Study of the life of plants.

1.8 Use of computers


The use of computers has become closely associated with quantitative techniques. With the evolution of more powerful computing techniques, users of these techniques are encouraged to explore new and more sophisticated methods of data analysis. Computers have the advantage of being a relatively inexpensive means of processing a large amount of data quickly and accurately. Computer data analysis packages such as SPSS, SAS and MINITAB have provided means for solving those problems which have long been quantifiable but computationally too complex or time-consuming for manual calculation. Problems which would take months to solve manually can be solved in a few minutes using computers.

UNIT 1 23
Basic mathematics for management

Summary
There is an ever-increasing demand for managers with numerate ability as well as literary skills, so that they can present numerate data and information which requires analysis and interpretation. More importantly, they can quickly scan and understand analysis provided both from within the firm and by outside organisations. In the competitive and dynamic business world, enterprises which are most likely to succeed, and indeed survive are those which are capable of maximising the use of the tools of management including quantitative techniques. This section has attempted to describe the meaning and use of various quantitative techniques in the field of business and management. The importance and complexity of the decision making process has resulted in the wide application of quantitative techniques in the diversified field of business and management. With the evolution of more powerful computing techniques, users of these techniques are encouraged to explore new and more sophisticated methods of data analysis. Quantitative approach in decision making however, does not totally eliminate the scope of qualitative or judgement ability of the decision maker.

Keywords
Descriptive models: Models which are used to describe the behaviour of a system based on data. Descriptive statistics: It is concerned with the analysis and synthesis of data so that better description of the situation can be made. Explanatory models: Models which are used to explain the behaviour of a system by establishing relationships between its various components. Inductive statistics: It is concerned with the developments of scientific criteria which can be used to derive information about the group of data by examining only a small portion (sample) of that group. Operations research: It is a scientific method of providing executive departments with a quantitative basis for decision regarding the operations under control. Predictive models: Models which are used to predict the future status of a system based on data. Quantitative techniques: It is the name given to the group of statistical and operations research (or programming) techniques. Statistical data: It refers to the numerical description of quantitative aspects of things. These descriptions may take the form of counts or measurement.

24 WAWASAN OPEN UNIVERSITY


BBM 511 Quantitative Techniques

Statistical decision theory: It is concerned with the establishment of rules and procedures for choosing the course of action from alternative courses of actions under uncertain situations. Statistical methods: These methods include all those devices of analysis and synthesis by means of which statistical data are systematically collected and used to explain or describe a given phenomenon.

Self assessment exercises


1. Think of any major decision you made recently. Recall the steps taken by you to arrive at the final decision. Prepare a list of those steps.

2. Comment on the following statements: a) Statistics are numerical statement of facts but all facts numerically stated are not statistics. b) Statistics is the science of averages.

3. What is the type of the following models? a) Frequency curves in statistics. b) Motion films. c) Flow chart in production control. d) Family of equations describing the structure of an atom.

4. List at least two applications of statistics in each functional area of management.

5. What factors in modern society contribute to the increasing importance of quantitative approach to management?

6. Describe the major phases of statistics. Formulate a business problem and analyse it by applying these phases.

UNIT 1 25
Basic mathematics for management

7. Explain the distinction between: a) Static and dynamic models. b) Analytical and simulation models. c) Descriptive and prescriptive models.

8. Describe the main features of the quantitative approach to management.

Further readings
Keller, G (2005) Statistics for Management and Economics, 7th edn, Mason, OH: Thomson Brooks/Cole. De Veaux, R D, Velleman, P F and Bock, D E (2008) Stats Data and Models, 2nd edn, Boston: Pearson Addison Wesley. Shenoy, G V, Srivastava, U K and Sharma, S C (1985) Quantitative Techniques for Managerial Decision Making, Eastern New Delhi: Wiley. Venkata, R K (1986) Management Science , Singapore: McGraw-Hill Book Company.

26 WAWASAN OPEN UNIVERSITY


BBM 511 Quantitative Techniques

Suggested answers to self assessment exercises

1. A sample list of steps in decision making: Observation A company makes specialised quality-testing equipment for food processing industry. The general manager has noticed that the number of late deliveries to customers seems to increase. Recognition After some time, the general manager receives a personal complaint from an important customer who suffered a second late delivery. The manager feels that the problem can no longer be ignored. Set objectives The general manager sees the problems as one of reducing the number of late deliveries. Understanding the problem The general manager calls the manufacturing, production and marketing managers to discuss the problem. The discussion is to identify the root of the problem. Determine options After identifying the cause of the problem, the next step is to consider and list out all possible options which are feasible. Evaluate options The group will evaluate each option in terms of quantity and quality. Select options The group will decide on the options or combination of options to be implemented. Monitor After setting the time frame, the new system is monitored and re-evaluated. If the results are not totally satisfactory, the problem is then redefined.

2. a) Statistics consist of numerical facts that can be collected, organised, evaluated and interpreted. However, not all facts numerically stated can be analysed effectively to generate useful information, for instance, the nominal qualitative data such as gender (0 for male and 1 for female). Thus, not all facts numerically stated are statistics. b) Statistics is the science of data, not only averages. It involves variability, skewness as well as categorical data.

UNIT 1 27
Basic mathematics for management

3. a) Frequency curves in statistics Descriptive model. b) Motion film Graphic model. c) Flow chart in production control Explanatory model. d) Family equations describing the structure of an atom Simulation model.

4. Marketing a) Analysis of marketing research information. b) Statistical records for building and maintaining an extensive market.

Production a) Production planning, control and analysis. b) Evaluation of machine performance.

Finance, accounting and investment a) Financial forecast and budget preparation. b) Financial investment decisions.

Personal a) Labour turnover rate. b) Employment trends.

Economics a) Measurement of gross national product and input output analysis. b) Determination of business cycle, long term-term growth and seasonal fluctuation.

Research and development a) Development of new product lines. b) Optimal use of resources.

Natural science a) Diagnosing the disease based on data.

28 WAWASAN OPEN UNIVERSITY


BBM 511 Quantitative Techniques

b) Judging the efficiency of a particular drug for curing a certain disease.

5. Due to the complexity of problems and scenarios faced by todays managers, quantitative methods can be used as the scientific approach towards the decision making. Quantitative tools and techniques provide means to define problems, compare alternative solutions and formulate hypothesis testing and prediction.

6. The major phases of statistics are data collection, data organisation, presentation, analysis and interpretation. For instance, to analyse problems associated with the usage of coal to generate electricity: Data collection: Historical data to help forecast future demand and specific data on the production of coal and electricity. Organising data: Develop a model that will forecast future demand and the potential environmental impact of various energy sources and usage. Presentation: Using the model to report on the investment in the new coal-energy system for a specific period of time. Analysis: The assumption of the model and the solution are carefully tested. Known data is used to make sure that the model is consistent with the current situation. This will also help to fine-tune the model and make it more accurate. Interpretation: Interpret the solutions and report the major finding. Also report on specific findings such as how coal should be handled and processed into energy to reduce pollution and negative environmental consequences. Findings will also serve in planning future energy management.

7. a) Static models are concerned with a particular set of fixed conditions and do not change in a short-term period. In a dynamic model, time plays an important role and there is an impact of change over a period of time. b) Analytical model consists of a mathematical structure and potential problems are solved using either mathematical or analytical technique. A simulation model is the experimentation on a mathematical structure of a real-life system. c) Descriptive model is a model used to describe the behaviour of a system based on data, for instance, regression models. Prescriptive models are

UNIT 1 29
Basic mathematics for management

models that provide the norms for the comparison of alternative solutions which will result in the selection of the best alternative, for instance, the linear programming models.

8. Quantitative approach to management is the scientific approach to managerial decision making that rules out whim, emotion and guesswork. The quantitative approach to management consists of defining a problem, developing a model, acquiring input data, developing a solution, testing the solution, analysing the results and implementing the results. Defining the problem This is the first step in the quantitative approach that develops a clear, concise statement of the problem. This statement will give direction and meaning to the subsequent steps. Developing a model Once the problem is selected, the next step is to develop a model which is usually a mathematical representation of a situation. Acquiring input data After developing a model, data must be collected to be used in the model. The collected data must be accurate so as to avoid misleading results. The collected data is known as input data. Developing a solution Developing a solution involves manipulating the model to arrive at the optimal solution to the problem. In some cases, it involves solving equations for the best solution. Testing the solution Before a solution can be analysed and implemented, it needs to be tested completely. Testing the solution requires the testing of both the input data and the model. Testing the input data and model is to determine the accuracy and completeness of the data used by the model. Analysing the results Analysing the results starts with determining the implication of the solution. The solution will result in some kind of action or change in the way an organisation is operating. The implications of these actions or changes must be determined and analysed before the results are implemented. Implementing the results This final step is to incorporate the solution into the company. After the solution is implemented, it should be closely monitored if it requires the analysis to be modified.

30 WAWASAN OPEN UNIVERSITY


BBM 511 Quantitative Techniques

UNIT 1 31
Basic mathematics for management

2 Functions and Progressions


Objectives
By the end of this section, you should be able to: 1. Identify the relationships that exist among business variables. 2. Define functional relationships. 3. Identify various types of functional relationships. 4. Use graphs to depict functional relationships. 5. Explain managerial applicability and the use of functional relationships in diverse fields. 6. Define and use the arithmetic and geometric progressions.

Introduction
For decision problems which use mathematical tools, the first requirement is to identify or formally define all significant interactions or relationships among primary factors (also called variables) relevant to the problem. These relationships are usually stated in the form of an equation (or set of equations) or inequations. Such type of simplified mathematical relationships help the decision maker in understanding (any) complex management problems. For example, the decision maker knows that the demand of an item is not only related to the price of that item but also to the price of the substitutes. Thus, if he can define a specific mathematical relationship (also called model) that may exist, then the demand of the item in the near future can be forecasted. The main objective of this section is to study mathematical relationships (or functions) in the context of managerial problems.

2.1 Definitions
Variable
A variable is something whose magnitude can vary or which can assume various values. The variables used in applied mathematics include sale, price, profit, cost, etc. Since magnitude of variables can vary, they are represented by symbols (such as x, y, z, etc.) instead of a specific number. In applied mathematics, a variable is represented by the first letter of its name, for example p for price or profit, q for quantity, c for cost, s for saving or sales, d for demand and so on. When we write x = 5, the variable takes a specific value.

32 WAWASAN OPEN UNIVERSITY


BBM 511 Quantitative Techniques

Variables can be classified in a number of ways. For example, a variable can be discrete (suspect to counting, e.g., 2 houses, 3 machines, etc.) or continuous (suspect to measurement, e.g., temperature, height., etc.).

Constant and parameter


A quantity that remains fixed in the context of a given problem or situation is called a constant. An absolute (or numerical) constant such as 2, , e, etc., retains the same value in all problems. An arbitrary (or parametric) constant or parameter retains the same value throughout any particular problem but may assume different values in different problems, such as wage rates of different category of labourers in an industrial unit. The absolute or numerical value of a constant b is denoted by |b| and means the magnitude of b regardless of its algebraic sign. Thus |b| = |b|.

Functions
We come across situations in which two or more variables are related to each other. For example, demand (D) of a commodity is related to its price (p). It can be mathematically expressed as D = f (p) (2-1)

This relationship is read as demand is function of price or simply f of p. It does not mean D equals f multiplies p. This mathematical relationship has two variables, D and p. These are called variables because they can take on different numerical values. Let us now consider a mathematical relationship that contains three variables. Assume that the demand (D) of a commodity is related to the price (p) per unit of the commodity, and the level of advertising expenditure (A). Then the general relationship among these variables can be expressed as D = f ( p , A) (2-2)

The functional notations of the type (2-1) and (2-2) are meant to give a general idea that certain variables are somehow related. However for making managerial decisions, we need a specific and explicit, not a general and implicit relationship among selected variables. For example, for the purpose of finding the value of demand (D), we make the general relationship (2-2) more specific as shown in (2-3). D = 4 + 3p 2pA + 2A2 (2-3)

UNIT 1 33
Basic mathematics for management

Now for any given values of p and A, the value of D can be calculated using the relationship (2-3). This means that the value of D depends on the values of p and A. Hence D is called the dependent variable and p and A are called independent variables. In this case, it may be noted that we have established a rule of correspondence between the dependent variable and independent variable(s). That is, as soon as values are assigned to the independent variable(s), the corresponding unique value for the dependent variable is determined by the given specific relationship. That is why a function is sometimes defined as a rule of correspondence between variables. The set of values given to an independent variable is called the domain of the function while the corresponding set of values of the dependent variable is called the range of the function. Other examples of functional relationships are as follows: 1. The distance (d) covered is a function of time (T) and speed (s), i.e., d = f (T, s). 2. Sales volume (v) of the commodity is a function of price (p), i.e., v = f (p). 3. Total inventory cost (T) is a function of order quantity (Q), i.e., T = f (Q). 4. The volume of the sphere (V) is a function of its radius (r), i.e., 4 V = f (r) or V = r3. 3 5. The extension (y) of a spring is proportional to the weight (w) (Hookes law), i.e., yw or y = kw where k is a constant. 6. The net present value (y) of an investment is a function of net cash flows (C) in different time periods, projects initial cash outlay (B), firms cost of capital (P) and the life of the project (N), i.e., y = f(C, B, P, N).

It is important to note that every mathematical relationship may not be a function. For example, consider the following relationship: y2 = x

It is not a function because corresponding to any value of x, the value of y is not unique. For example, when x = 4, y = +2 and 2.

34 WAWASAN OPEN UNIVERSITY


BBM 511 Quantitative Techniques

The dimension of a function is determined by the number of independent variables. For example: D = f (p) is a single-variable (or one-dimensional) function. D = f (p, A) is a two-variable (or two-dimensional) function. Y = f (Ct, B, P, N) is a multi-variable (or multi-dimensional) function.

In order to understand the nature of mathematical relationship (also called model) between independent variable(s) and dependent variable, we must be familiar with the terms such as parameter, constants and variables. Example 1 will illustrate the meaning of these terms.

Example 1 Suppose an industrial worker gets RM25 per day. If he works for 26 days in a particular month, then his total wage for that month is 25 26= RM650. If he works 25 days on certain months, then he would have earned RM625 per month. Thus, the total wages of the worker, assuming no overtime, can always be calculated as follows: Total wages = 25 number of days worked. If we let T = total wages D = number of days worked

then, T = 25 D.

This represents the relationship between total wages and the number of days worked. In general, the above relationship can also be written as: T = KD

where K is a constant for particular class of worker(s), to be assigned or determined in a specific situation. Since the value of K can vary for a specific situation, problem or context therefore it is called a parameter, whereas constants such as pi (denoted by ) which has an approximate value of 3.1416 and that remains the same from one problem context to another are called absolute constants. Quantities such as T and D which can assume various values in a given problem are called variables.

UNIT 1 35
Basic mathematics for management

Activity A
1. Find the domain and range of each of the following functions. 1 x1

a) y =

b) y = x ; y 0 c) y = 4 x ; y 0

2. Let 4p + 6q = 60 be an equation containing variables p (price) and q (quantity). Identify the meaningful domain and range for the given function when price is considered as an independent variable.

2.2 Types of function


In this section, some different types of functions are introduced which are particularly useful in calculus. 1. Linear functions A linear function is one in which the power of the independent variable is 1, the general expression of linear function having only one independent variable is: Y = f (x) = a + bx

where a and b are real numbers and x is an independent variable taking all numerical values in an interval. A function with only one independent variable is also called a single-variable function. Furthermore, a single variable function can be linear and non-linear. For example, y = 3 + 2x is a linear single-variable function, and y = 2 + 3x 5x2 + x3 is a non-linear single-variable function.

A linear function with one variable can always be graphed in a two dimensional plane (or space). This graph can always be plotted by giving different values to x and calculating corresponding values of y. The graph of such functions is always a straight line.

36 WAWASAN OPEN UNIVERSITY


BBM 511 Quantitative Techniques

Example 2 Plot the graph of the function, y = 3 + 2x. For plotting the graph of the given function, assign various values to x and then calculate the corresponding values of y as shown in the table below: x y 0 3 1 5 2 7 3 9 4 11 5 13

The graph of the given function is shown in Figure 1. y 13 11 9 7 5 3 0


Figure 1

y = 3 + 2x (3, 9) (2, 7) (1, 5) (0, 3) 1 2 3 4

(4, 11)

A function with more than one independent variable is defined, in general form, as: y = f(x1, x2, , xn) = ao + a1x1 + a2x2 + . . . + anxn

where a0, al, a2, ....., an are the given real numbers and x1, x2, ....., xn are independent variables taking all numerical value in the given intervals. Such functions are also called multivariable functions. A multivariable function can be linear and non-linear, for example, y = 2 + 3x1 + 5x2 is a linear multi-variable function, and y = 3 + 4x1 + 15x1x2 + 10x22 is a non-linear multivariable function.

UNIT 1 37
Basic mathematics for management

Multivariable functions may not be graphed easily because these require threedimensional plane or more dimensional plane for plotting the graph. In general, a function with n variables will require (n + 1) dimensional plane for plotting its graph.

2. Polynomial functions A function of the form y = f(x) = a1xn + a2xn1 + .. + anx + an+1 (2-4)

where ais (i = 1, 2, ..., n + 1) are real numbers, a1 0 and n is a positive integer is called a polynomial of degree n.

a) If n = 1, then the polynomial function is of degree 1 and is called a linear function. That is, for n = 1, function (2-4) can be written as: y = a1x1 +a2x (a1 0)

This is usually written as y = a + bx (Q x = 1)

where a and b symbolise a2 and a1 respectively.

b) If n = 2, then the polynomial function is of degree 2 and is called a quadratic function. That is, for n = 2, function (2-4) can be written as: y = a 1x 2 + a 2x 1 + a 3 (a1 0)

This is usually written as: y = ax2 + bx + c where a1 = a, a2 = b and a3 = c.

38 WAWASAN OPEN UNIVERSITY


BBM 511 Quantitative Techniques

3. Absolute value functions The functional relationship expressed by y = |x|

is known as an absolute value function, where |x| is known as the magnitude (or absolute value) of x. By absolute value we mean that whether x is positive or negative, its absolute value remains positive. For example |7| = 7 and |6| = 6.

Plotting the graph of the function y = |x|, assigning various values to x and then calculating the corresponding values of y, is shown in the table below: x y 3 3 2 2 1 1 0 0 1 1 2 2 3 3

The graph of the given function is shown in Figure 2.

y 5 y = x (3, 3) (2, 2) (1, 1) x 3 2 1 4 3 2 1 0 1 (1, 1) 2 3 x (2, 2) y=x (3, 3)

Figure 2

UNIT 1 39
Basic mathematics for management

4. Inverse function Take the function y = f(x). Then the value of y, can be uniquely determined for the given values of x as per the functional relationship. Sometimes, it is required to consider x as a function of y, so that for the given values of y, the value of x can be uniquely determined as per the functional relationship. This is called the inverse function and is also denoted by x = f 1(y). For example, consider the linear function: y = ax + b.

Expressing this in terms of x, we get yb a y b = cy + d a a 1 b and d = . a a

x=

where c =

This is also a linear function and is denoted by x = f 1(y).

5. Step function For different values of an independent variable x in an interval, the dependent variable y = f(x) takes a constant value, but takes different values in different intervals. In such cases the given function y = f(x) is called a step function. For example, y1, if 0 x < 50 y = y2, if 50 x < 100 y3, if 100 x < 150

The shape of the graph of this function is shown in Figure 3 for y3 < y2 < y1.

40 WAWASAN OPEN UNIVERSITY


BBM 511 Quantitative Techniques

y y1 y2 y3 x

50
Figure 3

100

150

6. Algebraic and transcendental functions Functions can also be classified with respect to the mathematical operations (addition, subtraction, multiplication, division, powers and roots) involved in the functional relationship between dependent variable and independent variable(s). When only the finite number of terms are involved in a functional relationship and variables are affected only by the mathematical operations, then the function is called an algebraic function, otherwise it is called transcendental function. The following functions are algebraic functions of x. a) y = 2x3 + 5xZ 3x + 9 b) y = x + 1 x2

c) y = x3

1 +2 x

The subclasses of transcendental functions are as follows:

a) Exponential function If the independent variable in any functional relationship appears as an exponent (or power), then that functional relationship is called exponential function, such as i) y = ax, a 1

ii) y = kax, a 1

UNIT 1 41
Basic mathematics for management

iii) y = kabx, a 1 iv) y = kex

where a, b, e and k are constants with a taking only a positive value. Such functions are useful for describing sharp increase or decrease in the value of dependent variable. For example, the exponential function y = kax curve rises to the right for a > 1, k > 0 and falls to the right for a < 1, k > 0 as shown in Figure 4(a) and Figure 4(b). y

y = kax, (a > 1, k > 0) k

0
Figure 4(a)

y = kax (a < 1, k > 0) x

Figure 4(b)

42 WAWASAN OPEN UNIVERSITY


BBM 511 Quantitative Techniques

b) Logarithmic functions A logarithmic function is expressed as y = loga x

where a > 0, a 1 is the base. It is read as `y is the log to the base a of x . This can also be written as x = ay

Thus from an exponential function y = ax, we may construct the logarithmic function x = logay by interchanging the variables. This shows that the inverse of an exponential function is a logarithmic function. The two most widely used bases for logarithms are 10 and e 2.7182. i) Common logarithm: It is the logarithm to the base 10 of a number x. It is written as log10x. If y = log10x, then x = 10 y. ii) Natural logarithm: It is the logarithm to the base e of a number x. It is written as loge x or ln x. When no base is mentioned, it will be understood that the base is e.

Some important properties of the logarithmic function y = log e x are as follows: i) log 1 = 0

ii) log e = 1 iii) log (xy) = log x + log y x iv) log ( ) = log x log y y v) log (xn) = n log x 1 log10e log10a log10e

vi) loge 10 =

vii) loge a = (log e10)(log10 a) =

viii) Logarithm of zero and negative number is not defined.

UNIT 1 43
Basic mathematics for management

Activity B
1. Draw the graph of the following functions: a) y = 3x 5 b) y = x2 c) y = log2 x

2. The data of machine operating cost (c) and the age (t) of the machine art is shown in the following table: t (years) c (in RM 000) 1 5 2 8 3 13 4 20 5 29

a) Express operating cost as a function of the machine age. b) Sketch the graph of the function derived in (a).

2.3 Solution of functions


The value(s) of x at which the given function f(x) becomes equal to zero are called the roots (or zeros) of the function f(x). For the linear function y = ax + b

the roots are given by ax + b = 0 b x= . a b Thus, if x = is substituted in the given linear function y = ax + b then it becomes a equal to zero. or

44 WAWASAN OPEN UNIVERSITY


BBM 511 Quantitative Techniques

In the case of quadratic function y = ax2 + bx + c,

we have to solve the equation ax2 + bx + c = 0, namely to find the roots of y given a 0. The general value of x for which the given quadratic function will become zero is given by b b2 4ac where a 0. 2a

x=

Thus, in general, there are two values of x for which y becomes zero. One value is b + b2 4ac 2a

x=

and the other value is b b2 4ac . 2a

x=

It is very important to note that the number of roots in the given function is always equal to the highest power of the independent variable.

Particular cases: The expression b2 4ac in the above formula is known as discriminant which determines the nature of the roots as discussed below: 1. If b2 4ac > 0, then the two roots are real and unequal. b . 2a

2. If b2 4ac = 0 or b2 = 4ac, then the two roots are equal to

3. If b2 4ac < 0, then the two roots are imaginary (not-real) because of the square root of a negative number. The roots of a polynomial of the form: y = (x a)(x b)(x c)(x d) are a, b, c, d, .... .

UNIT 1 45
Basic mathematics for management

Activity C
Given that f(x) = (x 4)(x + 3), then find: 1. f(4), f(1) and f(3). 2. Roots of the function. _______________________________________________________________ _______________________________________________________________ _______________________________________________________________ _______________________________________________________________

2.4 Business applications


We often talk of supply and demand functions, cost functions, profit functions, revenue functions, production functions, utility functions, etc., in applied mathematics. In this section, a few examples are given by constructing such functions and obtaining their solutions.

Example 3 (Linear functions) A company sells x units of an item each day at the rate of RM50 per unit. The cost of manufacturing and selling these units is RM35 per unit plus a fixed daily overhead cost of RM1000. Determine the profit function. How would you interpret the situation if the company manufactures and sells 400 units of the items a day?

Solution The total revenue received by the company per day is given by: Total revenue (R) = (price per unit) (number of items sold) = 50x.

The total cost of manufactured items per day is given by: Total cost (c) = (Variable cost per unit) (number of items manufactured) + (fixed daily overhead cost) = 35x + 1000.

Thus, total profit (p) = (Total revenue) (Total cost) = 50x (35x + 1000) =15x 1000.

46 WAWASAN OPEN UNIVERSITY


BBM 511 Quantitative Techniques

If 400 units of the item are manufactured and sold, then the profit is given by: p = 15 400 1000 = 400.

The negative profit indicates loss. Thus, if the company manufactures and sells 400 units of the item, it would incur a loss of RM400 per day.

Example 4 (Quadratic functions) Let the market supply function of an item be q =160 + 8p, where q denotes the quantity supplied and p denotes the market price. The unit cost of production is RM4. It is felt that the total profit should be RM500. What should be the market price for the item to achieve this profit?

Solution Total profit function can be constructed as follows: Total profit (P) = total revenue total cost = (price per unit quantity supplied) (cost per unit quantity supplied) = p.q c.q = (p c).q

Given that c = RM 4 and q = 160 + 8p. Then total profit function becomes P = (p 4)(160 + 8p) = 8p2 + 128p 640.

If P = 500, then we have 500 = 8p2 + 128p 640 8p2 + 128p 1140 = 0 Qp = or

128 (128)2 4 8 (1140) 28 128 229.92 = 16 = 6.36 or 22.37.

Since negative price has no economic meaning, therefore the required price per unit should be RM6.36.

UNIT 1 47
Basic mathematics for management

Activity D
1. Consider the quadratic equation 2x2 8x + c = 0. What is the value of c, if the equation has a) real roots? b) equal roots? c) imaginary roots?

2. A newsboy buys papers for the price of p1 cents per paper and sells them at a price of p2 cents per paper (p2>p1). The unsold papers at the end of the day, are bought by a wastepaper dealer for p3 price per paper(p3 < p1). a) Construct the profit function of the newsboy. b) Construct the opportunity loss function of the newsboy.

2.5 Sequence and series


Sequence
If for every positive integer n, there is a corresponding number an such that an is related to n by some rules, then the terms a1, a2, ..., an, ... are said to form a sequence. A sequence is denoted by bracketing its nth term, i.e., (an) or {an}. Examples of a few sequences are: 1. If an = n2, then sequence {an} is 1, 4, 9, 16, ..., n2, ... 1 1 1 1 1 2. If an = , then sequence {an} is 1, , , ..., , ... n 2 3 4 n n2 1 4 9 n2 , then sequence {an} is , , , ..., , ... n+1 2 3 4 n+1

3. If an =

The concept of sequence is very useful in finance. Some of the major areas which play a vital role are instalment buying, simple and compound interest problems, annuities and their present values, mortgage payments and so on.

48 WAWASAN OPEN UNIVERSITY


BBM 511 Quantitative Techniques

Series
A series is formed by connecting the terms of a sequence with plus or minus sign. Thus if an is the nth term of a sequence, then a1 + a2 + ... + an is a series of n terms.

2.6 Arithmetic progression (AP)


A progression is a sequence whose successive terms indicate the growth or progress of some characteristics. An arithmetic progression is a sequence whose term increases or decreases by a constant number called common difference of an AP and is denoted by d. In other words, each term of the arithmetic progression after the first is obtained by adding a constant d to the preceding term. The standard form of an AP is written as a, a + d, a + 2d, a + 3d, ... where a is called the first term. Thus the corresponding standard form of an arithmetic series becomes a + (a + d) + (a + 2d) + (a + 3d) + ...

Example 5 Suppose we invest RM100 at a simple interest of 15% per annum for 5 years. The amount at the end of each year is given by 115, 130, 145, 160, 175. This forms an arithmetic progression.

The nth term of an AP


The nth term of an AP is also called the general term of the standard AP. It is given by Tn = a + (n 1) d for n =1, 2, 3, ...

Sum of the first n terms of an AP


Consider the first n terms of an AP, a, a + d, a + 2d, a + 3d, ..., a + (n 1)d

UNIT 1 49
Basic mathematics for management

The sum, Sn of these terms is given by sn = a + (a + d) + (a + 2d) + (a + 3d) + + a + (n 1)d = (a + a + + a) + d{1 + 2 + 3 + + (n 1)} = n.a + d{ n(n 1) } 2 (using formula for the sum of the first (n 1) natural numbers)

n {2a + (n 1)d} 2

Example 6 Suppose Mr. X repays a loan of RM3250 by paying RM20 in the first month and then increases the payment by RM15 every month. How long will it take to clear his loan?

Solution Since Mr. X increases the monthly payment by a constant amount, RM15 every month, therefore d = 15 and the first month instalment is, a = RM20. This forms an AP. Now if the entire amount is paid in n monthly instalments, then we have n {2a + (n 1)d} 2

sn =

or

3250 =

n {2 20 + (n 1)15} 2

6500 = n{25 + 15n} 15n2 = 25n 6500 = 0.

This is a quadratic equation in n. Thus to find the values of n which satisfy this equation, we shall apply the following formula as discussed before. b b2 4ac 25 (25)2 4 15 (6500) = 2a 2 15 = 25 625 = 20 or 21.66. 30

x=

The value, n = 21.66 is meaningless as n is a positive integer. Hence, Mr. X will pay the entire amount in 20 months.

50 WAWASAN OPEN UNIVERSITY


BBM 511 Quantitative Techniques

Activity E
1. Find the 15th term of an AP whose first term is 12 and the common difference is 2. 2. A firm produces 1500 TV sets during its first year. If the total production of the firm at the end of the 15th year is 8300 TV sets, then: a) Estimate the number of units if production has increased consistently in each year. b) Based on the estimate of the annual increment in production, forecast the amount of production for the 10th year.

2.7 Geometric progression (GP)


A geometric progression (GP) is a sequence where each term increases or decreases by a constant ratio called common ratio of GP and is denoted by r. In other words, each term of GP is obtained after the first by multiplying the preceding term by a constant r. The standard form of a GP is written as a, ar, ar2, where a is called the first term. Thus the corresponding geometric series in standard form becomes a + ar + ar2 + ...

Example 7 Suppose we invest RM100 at a compound interest of 12% per annum for three years. The amount at the end of each year is calculated as follows: 1. Interest at the end of first year = 100 12 = RM12. 100 12 ) 100

Amount at the end of first year = Principal + Interest = 100 + 100 (

= 100 1 +

12 . 100 12 ) at the 100

This shows that the principal of RM100 becomes RM100 (1 + end of first year.

UNIT 1 51
Basic mathematics for management

2. Amount at the end of second year = (Principal at the beginning of second year) {1 + = 100 {1 + 12 12 } {1 + } 100 100 12 2 }. 100 12 2 12 } {1 + } 100 100 12 3 }. 100 12 } 100

= 100 {1 +

3. Amount at the end of third year = 100 {1 +

= 100 {1 +

Thus, the progression giving the amount at the end of each year is 12 12 2 12 3 }; 100 {1 + } ; 100 {1 + };. 100 100 100 12 ). 100

= 100 {1 +

This is a GP with common ratio r = (1 +

In general, if P is the principal and i is the compound interest rate per annum, then the amount at the end of first year becomes P(1 + of successive years forms a GP. P(1 + i i 2 ); P(1 + ); 100 100 i ). 100 i ). Also the amount at the end 100

with r = (1 +

The nth term of GP


The nth term of GP is also called the general term of the standard GP. It is given by Tn = ar n1, n = 1, 2, 3, ...

It may be noted here that the power of r is one less than the index of Tn, which denotes the rank of this term in the progression.

52 WAWASAN OPEN UNIVERSITY


BBM 511 Quantitative Techniques

Sum of the first n terms in GP


Consider the first n terms of the standard form of GP a, ar, ar2, , ar n1.

The sum, Sn of these terms is given by Sn = a + ar + ar2 + ... + ar n2 + ar n1 (2-4)

Multiplying both sides by r, we get rSn = ar + ar2 + ar3 + ... + ar n1 + arn (2-5)

Subtracting (2.5) from (2.4), we have Sn rSn = a arn Sn (1 r) = a(l rn) or

Sn =

a(l rn) ; r 1 and < 1. (1 r)

Changing the sign of the numerator and denominator, we have a(rn 1) , r 1 and > 1 (r 1)

Sn =

1. If r = 1, GP becomes a, a, a, .... so that Sn in this case is Sn = n.a. 2. If the number of terms in a GP are infinite, then Sn = a , |r| < 1. 1r

For r 1, the sum approaches infinity.

UNIT 1 53
Basic mathematics for management

Example 8 A car is purchased for RM80,000. Depreciation is calculated at 5% per annum for the first 3 years and 10% per annum for the next 3 years. Find the money (book) value of the car after a period of 6 years.

Solution
1. Depreciation for the first year = 80,000 5 . 100

Thus the depreciated value of the car at the end of first year is: = 80,000 80,000

5 100

= 80,000 1

5 . 100

2. Depreciation for the second year = (Depreciated value at the end of first year) (Rate of depreciation for second year) = 80,000 1

( ( ( (

5 5 . 100 100

)( ) ) )( )

Thus the depreciated value at the end of the second year is = (Depreciated value after first year) (Depreciation for second year) = 80,000 1 5 5 80,000 1 100 100 5 5 1 100 100 5 2 . 100

)( )
5 100

= 80,000 1

= 80,000 1

Calculating in the same way, the depreciated value at the end of three years = 80,000 1

5 3 . 100

54 WAWASAN OPEN UNIVERSITY


BBM 511 Quantitative Techniques

3. Depreciation for the fourth year = 80,000 1

5 100

)( )
3

10 . 100

Thus the depreciated value at the end of the fourth year is = (Depreciated value after third year) (Depreciation value for fourth year) = 80,000 1

= 80,000 1

( ( (

5 3 5 80,000 1 100 100 5 100


3

) )( )(
3

)( )
3

10 100

10 . 100

) )

Calculating in the same way, the depreciated value at the end of six years becomes = 80,000 1 5 100 1 10 100
3

= RM49,980.24.

Activity F
1. Determine the common ratio of the GP 1 1 49, 7, 1, , , . 7 49 a. Find the sum of the first 20 terms of GP. b. Find the sum to infinity of the terms of GP.

2. The population of a country in 1985 was 50 million. Calculate the population in the year 2000 if the compounded annual rate of increase is (a) 1% (b) 2%.

UNIT 1 55
Basic mathematics for management

Summary
The objective of this section is to provide you an exposure to functional relationship among decision variables. We started with the mathematical concept of functions and defined terms such as constant, parameter, independent and dependent variable. Various examples of functional relationships are mentioned to see the concept in broad perspective. Various types of functions which are normally used in managerial decision making are enumerated along with suitable examples, their graphs and solution procedure. Finally, the applications of functional relationships are demonstrated through several examples. Attention is then directed to defining the arithmetic and geometric progressions and subsequently to their applications.

Keywords
Arithmetic Progression (AP): An AP is a sequence whose terms increases or decreases by a constant number. Algebraic and transcendental function: When only finite number of terms are involved in a functional relationship and variables are affected only by the mathematical operations, then the functions are called algebraic functions, otherwise they are called transcendental functions. Constant: A quantity that remains fixed in the context of a given problem or situation. Exponential function: If the independent variable in any functional relationship appears as an exponent (or power), then such functional relationship is called exponential function. Function: It is the rule of correspondence between dependent variable and independent variable(s) so that for every assigned value to the independent variable, the corresponding unique value for the dependent variable is determined. Geometric Progression (GP): A GP is a sequence whose terms increase or decrease by a constant ratio. Linear function: A function whose graph is a straight line is called a linear function. Logarithmic function: The inverse of exponential function is called a logarithmic function. Parameter: A quantity that retains the same value throughout any particular problem but may assume different values in different problems.

56 WAWASAN OPEN UNIVERSITY


BBM 511 Quantitative Techniques

Polynomial function: A function of degree n is called a polynomial function of degree n. Series: A series is formed by connecting the terms of a sequence with plus or minus sign. Sequence: If for any positive integer n, there corresponds a number an such that an is related to n by some rules, then the terms al, a2, ..., an are said to form a sequence. Step function: If for values of an independent variable, the dependent variable takes a constant value in different intervals, then the function is called step function. Variable: A quantity that can assume various values.

Further readings
1. Tan, S T (2007) Applied Mathematics for the Managerial, Life and Social Sciences, 4th edn, Belmont, CA: Thomson Brooks/Cole. 2. Haeussler, E F, Paul R S and Wood, R J (2005) Introductory Mathematical Analysis for Business, Economics and the Life and Social Sciences, 11th edn, Upper Saddle River, New Jersey: Pearson Prentice Hall. 3. Lial, M L, Hungerford, T W and Holcomb, J P (2007) Mathematics with Applications in the Management, Natural and Social Sciences, 9th edn, Boston: Pearson Addison Wesley.

UNIT 1 57
Basic mathematics for management

Suggested answers to activities

Activity A
1. a) The domain x is any real number except 1 as any arithmetic expression cannot be divided by 0. The range y is any real number except 0. b) The domain is x = 0 given that y 0 as y 0 for any x 0. c) The domain is x 4, the range is y 0 .

2. 4p + 6q = 60 4p = 60 6q 3 p = 15 q. 2 3 3 The domain is p 0, the range is 15 q 0 q 15 q 10. 2 2

Activity B
1. a) 12 10 8 6 4 2 0 2 4 6 0 1 2 3 4 5 6 x y

58 WAWASAN OPEN UNIVERSITY


BBM 511 Quantitative Techniques

b) 30 25 20 15 10 5 0

c) 3.5 3.0 2.5 2.0 1.5 1.0 0.5 0

10 11

2. a) The function of the machine age in operating costs is non-linear. As the increase of operating costs from year to year follows arithmetic progression, the quadratic function is a good fitted function to be used. Function used is c = at2 + bt + c, where a, b and c are constants. Substitute any 3 sets of values from the table. When t = 1, c = 5; 5=a+b+c (1)

UNIT 1 59
Basic mathematics for management

When t = 2, c = 8; 8 = 4a + 2b + c (2)

When t = 3, c = 13 13 = 9a + 3b + c (3)

(2) (1) 3 = 3a + b b = 3 3a

(4)

(3) (1) 8 = 8a + 2b (5)

Substitute (4) into (5) 8 = 8a + 2(3 3a) 8 = 8a + 6 6a a=1

From (4) b = 3(1) 3 =0

From (1) 5=1+0+c c = 4.

Hence the required function is c = t 2 + 4.

60 WAWASAN OPEN UNIVERSITY


BBM 511 Quantitative Techniques

2. b) The graph is a quadratic graph.

C (RM 000) 35 30 Thousands 25 20 15 10 5 0 0 1 2 3 4 5 6 t (years)

Activity C
1. f(x) = (x 4)(x + 3) f(4) = (4 4)(4 + 3) = 0. f(1) = (1 4)(1 + 3) f(4) = 5 2 = 10. f(3) = (3 4)(3 + 3) = 0.

2. The roots of the function are given by: (x 4)(x + 3) = 0 (x 4) = 0 x=4

or or

(x + 3) = 0 x = 3.

UNIT 1 61
Basic mathematics for management

Activity D
1. 2x2 8x + c = 0 a) If the equation has real roots: b2 4ac 0 (8)2 4 2 c 0 64 8c 0 8c 64 c8

b) If the equation has equal roots: b2 4ac = 0 (8) 4 2 c = 0 64 8c = 0 8c = 64 c=8


2

c) If the equation has imaginary roots: b2 4ac < 0 (8)2 4 2 c < 0 64 8c < 0 8c > 64 c>8

2. Let x = amount of paper bought, y = amount of paper sold. a) Profit function = yp2 + (x y)p3 xp1. b) Opportunity loss function due to overstock = (p2 p3 )(x y).

Activity E
1. T15 = 12 + (15 1)2 = 12 + 48 = 40

62 WAWASAN OPEN UNIVERSITY


BBM 511 Quantitative Techniques

2. a) The total production at the end of the 15th year is 8300. T15 = 1500 + (15 1)d 8300 = 1500 + 14d d= 8300 1500 14

= 485.7 486 sets of TV.

b) The forecast amount of production for the 10th year is: T10 = 1500 + (10 1)486 = 1500 + (9 486) = 5871 sets of TV.

Activity F
1. The common ratio, r of the GP is: r= 7 49 1 7

a) The sum of the first 20 terms is

s20 =

49 1

( )) 1 ( )
1
20

1 7

49
6 7

()
1 7

18

57.17.

UNIT 1 63
Basic mathematics for management

b) The sum to infinity is s = 49 1 7 = 49


6 7 1

57.17.

2. a) For the year 1985, population = 50. The annual compound rate is 0.01. For the year 1986, population = 50(0.01) + 50 = 50(1 + 0.01) = 50(1.01). For the year 1987, population = 50(1.01)(0.01) + 50(1.01) = 50(1.01)2. Hence for the year 2000, population = 50(1.01)20001985 = 50(1.01)15 = 58.05 58 (in millions).

b) For the year 1985, population = 50. The annual compound rate is 0.02. For the year 1986, population = 50(0.02) + 50 = 50(1 + 0.02) = 50(1.02). For the year 1987, population = 50(1.02)(0.02) + 50(1.02) = 50(1.02)2. Hence for the year 2000, population = 50(1.02)20001985 = 50(1.02)15 = 67.29 67 (in millions).

64 WAWASAN OPEN UNIVERSITY


BBM 511 Quantitative Techniques

UNIT 1 65
Basic mathematics for management

3 Basic Calculus and Its Applications


Objectives
By the end of this section, you should be able to: 1. Define the meaning of the term calculus and its branches. 2. Apply the concepts of limit and slope which are fundamental to the understanding of calculus. 3. Define differential calculus. 4. Identify the type of decision problems which can be solved with the help of differential calculus.

Introduction
In the past, the term calculus as a branch of mathematics was familiar only to scientists. The managers and students of business management were a little concerned about its usefulness. But, with the increasing need of quantitative techniques in the solution of business problems, there is a growing tendency to use quantitative techniques based on calculus in the solution of business problems. Calculus-based techniques are extensively used in economics, operations management, marketing, financial management, etc. Calculus is particularly useful in those situations where we are interested in estimating the rate at which things change. For example, it has a role to play when we are interested in knowing how the sales volume or sales is affected when the prices change or how the total cost, price, etc., are affected when the volume of output changes. There are two branches of calculus: differential calculus and integral calculus. These two terms are the reverse of each other, as are addition and subtraction, and multiplication and division. Differential calculus is concerned with determining the rate of change of a given function due to a unit of change in one of the independent variables while integral calculus is concerned with the inverse problem of finding a function when its rate of change is given. This cannot be illustrated with real examples because integral calculus is beyond the scope of this section. In this section, we will only be concerned with differential calculus. Analysis in business and economics is frequently concerned with change, therefore differential calculus should find wide applications in business. Marginal analysis in economics is perhaps the most direct application of differential calculus in business. Business problems concerned with things such as maximisation of profits and minimisation of costs, under various assumptions, can be solved using differential calculus. The objective of this section is to give you an idea about the rate of change

66 WAWASAN OPEN UNIVERSITY


BBM 511 Quantitative Techniques

of a function. The applications of this concept to marginal analysis and to various problems of maximisation and minimisation are discussed in this section.

3.1 Limit and continuity


Limit
Sometimes, we wish to determine the behaviour of a function y = f (x) as the independent variable x approaches some particular value, say a. For example, it may be interesting to know limiting saturation level of sales as advertising efforts are increased. The formal definition of limit may look a little abstract, therefore the notion of limit of a function is easier to understand in an intuitive sense. Consider a function f(x) defined as: f(x) = x 1.

Now as the value of x approaches 1, the value of the function f (x) becomes smaller and closer to zero. This phenomenon of x approaches a is symbolically written as x a. The corresponding value of f(x), say `L as x a is called the limit of the function, and it is symbolically written as:
xa

Limit f(x) = L or xLt. f(x) = L a

or f(x) L as x a.

Example 1 If f(x) = 2x + 5, then xLt. f(x) = 5. It can be illustrated as shown below: 0

x 2 1 1 2 1 5 1 10

y = f (x) = 2x + 5 9 7 6 27 5 26 5

UNIT 1 67
Basic mathematics for management

x 1 100 1 1000

y = f (x) = 2x + 5 251 50 2501 500

Alternatively, symbolical notations of the limit of the given function when we allow x to take different values are as follows: Notations 1.
x

Example 1 Lt. (1 + ) = 1 x

Lt. f(x) = L

or f(x) L as x 2.
x

Lt. f(x) = L

or f(x) L as x 3.
x

1 Lt. (1 )=1 x x Lt. (x3 8) = , and also Lt. (x3 8) = 1 2} = (x 2)

Lt. f (x) = , and Lt. f (x) =

4.

xa

Lt. f(x) =

or f(x) as x a

x2

Lt. {

There may be certain situations where limit takes the meaningless form such as 0 0 , , 0 and . Such forms are also called indeterminate forms. In all such cases, 0 the given functions are simplified to obtain a determinate value.

Example 2 If f (x) = x2 4 , then find the limit of f (x) as x 2. x2

68 WAWASAN OPEN UNIVERSITY


BBM 511 Quantitative Techniques

Solution f (x) = x2 4 (x 2) (x + 2) = x2 x2

For x 2, x 2 0, then
x2

Lt. f (x) = x Lt.2 (x + 2) = 4

However, at x = 2, f (x) =

44 0 = (an indeterminate form). 22 0

It may be noted that the limit of the given function as x 2 is not the value of the function when x = 2. The limit of the function is 4 whereas the value is indeterminate.

Rules of limit of a function


From the definition of limits, it is now easy to derive some basic results in the operation of limits. Suppose there are two functions f (x) and g(x) having
xa

Lt. f (x) = L1

and
xa

Lt. g (x) = L 2

then 1. The limit of a sum (or difference) of two functions is equal to the sum (or difference) of the limits of the two functions. That is
xa

Lt. { f (x) g(x)} = xLt. f (x) x Lt.a g(x) a


= L1 L2

2. The limit of the product of two functions is equal to the product of limit of functions.
xa

Lt. { f (x) g(x)} = xLt. f (x) xLt. g(x) a a


= L1 L2

UNIT 1 69
Basic mathematics for management

3. The limit of the quotient of two functions is equal to the quotient of their limits, provided the limit of the divisor is not zero.

Lt. x a

{ }

f (x) L f (x) xLt. = a = 1 , provided L2 0 g(x) Lt. g(x) L2


xa

4. The limit of a constant is equal to that constant


xa

Lt. K = K

5. The limit of the nth power of any function is equal to the nth power of the limit of the function.

f (x) Lt. { f (x)}n = xLt. x a a


= {L1}n

The limit of exponential function Suppose a function is defined as: f(n) = 1 +

( )

1n . n

Then 1+ Lt. f (n) =x Lt. x

( )

1 n . n

= e (= 2.71828).

Also, for every real number x, we have e x = xLt. 1+

( )

x n . n

70 WAWASAN OPEN UNIVERSITY


BBM 511 Quantitative Techniques

Example 3 A sum of RM P is initially invested at the rate of r per annum, to be compounded annually. Then the compound value of money at the end of n years is given by A = P (l + r)n.

But if the interest is compounded more than once a year, then we have A = P (l + r mn ) m

where m is the number of times per year where compounding occurs. That is, the interest is compounded at intervals of 1 years. m

If m (the interest is compounded at a very small interval), then we have Lt. A = xLt. P l+ x

[( ) ]
r m

m n

= P(er)n = Pern .

Hence, a sum of RM P invested initially at the rate of r per annum, to be compounded continuously becomes A = P . ern at the end of n years.

Activity A
1. Evaluate:

a) nLt. 1+

( )
1 n

b)nLt.

( )
n2 n+1 S = 2000 + 4000{1 e(0.01)x }.

2. The sales S (in RM 000) of a product as a function of advertising expenditure x is given by:

Find the limit of S as x and interpret your result.

UNIT 1 71
Basic mathematics for management

Continuity
A function y = f(x) is said to be continuous at a point x = a if : 1. f(a) exists (or defined). 2. xLt. f(x) exists. a

3. xLt. f(x) = f(a). a

Condition (3) implies that both the right hand limit and left hand limit should exist and be equal to the value of the function at x = a. That is, limit of f(x) in the neighbourhood (close to) of x = a (or at x = a + h and x = a h where h 0) should exist. The limit is said to exist if its value is finite. For example, if Lt. f(x) = as x a, then this means f(x) becomes arbitrarily large as x approaches a. It should be remembered that is not a number. A function f(x) is said to be continuous in (or on) an open interval (b, c) or closed interval [b, c] if it is continuous at each and every point of the interval. Otherwise it is said to be discontinuous. From this definition of continuity, it follows that the graph of a function that is continuous in (or on) an interval consists of unbroken curve (i.e., a curve that can be drawn without raising the pen from the paper) over that interval as shown in Figure 5(a) and 5(b).

(a) Discontinuous function y 400 400 300 200 100 x 300 200 100 y

(b) Continuous function

0
Figure 5

10

20

30

40

10

20

30

40

72 WAWASAN OPEN UNIVERSITY


BBM 511 Quantitative Techniques

Example 4 Discuss the nature of the following functions. 1. f(x) = 1 at x = 2. x2

2. f(x) = x2 at x = 2.

Solution 1. The function y = 1 = 0 1 is discontinuous at x = 2 because x2

f(2) =

i.e., the function is not defined for x = 2 because it does not have finite value.

2. f(2) = (2)2 = 4 (finite value). Also Right Hand Limit (R.H.L.) = Lt. (2 + h)2 = Lt. (4 + h2 + 4h) = 4 (finite). h 0 h 0

Left Hand Limit (L.H.L.) = hLt. (2 h)2 = hLt. (4 + h2 + 4h) = 4 (finite). 0 0


Since all the conditions of continuity are satisfied, therefore the function is continuous.

Activity B
The total cost of purchasing x units of an item within each interval, c(x) is as follows: 15x; 0 x 200 c(x) = 13x; 200 < x 400 10x; x > 400

Find the points of discontinuity.

UNIT 1 73
Basic mathematics for management

3.2 Concept of slope and rate of change


The term slope is used to measure the degree of steepness or rate of change of a function. In general, it is defined as the change in the dependent variable caused by one unit of change in one of the independent variables. The slope is denoted by m or tan ( is the angle of inclination of the given line with x-axis).

Slope of a straight line


Consider the case of the total cost of producing an item. Usually the total cost of production is a function of the fixed (set-up) cost plus a constant additional cost for each item produced. If the fixed cost is RM3 and additional cost is RM1.50, then the total cost, y is represented by y = 3 + 1.5x where x is the number of items produced. Clearly, x is the independent variable and y is the dependent variable. This equation has been graphed in Figure 6. It represents a straight line.

y Total cost y = 3 + 1.5x 6


rise

B(x2, y2)

y x

5 A(x , y ) 1 1 4 3 2

y = y2 y1

x = x2 x1 run

y2

y1 x2

x1 0 1 2 3 4 5 6 x Number produced

Figure 6

74 WAWASAN OPEN UNIVERSITY


BBM 511 Quantitative Techniques

Consider two points A and B on the line whose coordinates are (x1, y1) and (x2, y2) respectively. Suppose, we employ the symbol (delta) to indicate a very small change in the value of a variable or quantity. This change can be a positive or a negative change. If x represents the change (or increment) in the value of x and y represents the change in the value of y due to the change in x, then the ratio

( )
y x

of the change in the dependent variable y due to one unit of change in the independent variable, is called the slope and is defined as m = tan = rise run

or

y y2 y1 7.5 4.5 = = x x2 x1 31 = 1.5 (coefficient of x).

Thus, in the case of straight line relationship which we are currently considering, the slope is simply given by the coefficient of the independent variable. In this case, the slope is +1.5 (the plus sign indicates that y increases when x increases and vice-versa). Consider the equation of the line y = 3 or 3 + 0x (i.e., cost of production is independent of the number of items produced). It is obvious that the term involving x has a coefficient of zero. That is, the slope of this line is zero and hence it is a horizontal line as shown in Figure 6. It should be noted that the slope (rate of change) of a line remains constant at all points on the line, i.e., rate of change of y as x changes is constant throughout the length of the line. However, the slope of a curve (i.e., a non-linear function) changes from point to point and thus the slope must be determined for each particular point of interest.

Positive and negative slopes


The slope +1.5 in the case just discussed is an example of a positive slope which indicates that dependent variable y increases (or decreases) as independent variable x increases (or decreases). But if the value of dependent variable y decreases as independent variable x increases and vice-versa, then the slope is always negative. For example, let the sales of an item be the function of the price charged, and the exact relationship between these two is given by y = 100 5x. In this case, the slope is 5 (negative) which indicates that sales, y decreases with increasing values of price, x and vice-versa.

UNIT 1 75
Basic mathematics for management

Activity C
Suppose a salesman is paid a fixed sum of RM500 per month together with a bonus of RM2 for all items sold. Devise functional relationship for his salary and determine the slope of the line.

Slope of a curve (at a point)


For non-linear functions, the slope changes from point to point. Thus, it is necessary to specify the point at which the slope is to be determined. The procedure for computing the slope in this case is also the same as in the case of the straight line. This means that we must compute the ratio y at a specified point. Suppose the total x

cost of the stock of an item, y as a function of order quantity x is represented as: y = 4x + 200 . x

This equation has been graphed in Figure 7. It represents a curve, as follows.

Total cost, y

B y = 1.34 x A

y = 3.94 x

10

15

20

25

Quantity ordered, x

Figure 7

76 WAWASAN OPEN UNIVERSITY


BBM 511 Quantitative Techniques

For x = 5, y = 20 +

200 = 60 5 200 = 56.66. 7.5

x = 7.5, y = 30 +

Between x = 5 and x = 7.5, we have y 56.66 60 = = 1.34. x 7.5 5

For x = 20, y = 80 +

200 = 90 20 200 = 98.88. 22.5

x = 22.5, y = 90 +

Between x = 20 and x = 22.5, we have y 98.88 90 = = 3.94. x 22.5 20

From these two values, it is clear that the slope of a curve is different at different points, and the absolute value of the ratio to the absolute value of the ratio y in the first case is smaller as compared x

y in the second case. This shows that the value of x

y is much more sensitive to changes in the lower range of x. The negative slope between x = 5 and 7.5 indicates that the total stock holding cost decreases as the size of order increases on this part of the curve. Whereas between x = 20 and x = 22.5, the stock holding cost increases as the size of order increases on this part of the curve.

Activity D
Suppose, total cost, y of the stock of an item as a function of order size, x is represented by the equation 200 . x

y = 4x +

UNIT 1 77
Basic mathematics for management

Compare the slope between x = 8 and 9 and x = 20 and 21. Interpret your results. ____________________________________________________________________ ____________________________________________________________________ ____________________________________________________________________ ____________________________________________________________________ ____________________________________________________________________ ____________________________________________________________________ ____________________________________________________________________

3.3 Concept of derivative


The term derivative is a generalised expression for measuring the rate of change or slope of a function. Supposing A and B are two points on the curve (Figure 8) whose coordinates are (x1, yl) and (x2, y2) respectively. y y2 C D y1 A(x1, y1) Tangent to curve at point A B(x2, y2)

y = y2 y1

x = x2 x1 0
Figure 8

x1

x2

In Figure 8, the average slope of the curve between the two points A and B is measured by the slope of the line joining the points A and B. That is, slope of the line AB = y2 y1 y = x2 x1 x (3.1)

Assuming that the mathematical equation of the curve in the figure is represented by y = f(x).

78 WAWASAN OPEN UNIVERSITY


BBM 511 Quantitative Techniques

Then y1 = the value of f(x) at x = x1 = f(x1). Similarly y2 = f(x2).

Substituting for y1 and y2 in equation (3.1), we have y f(x2) f(x1) = x x2 x1

(3.2)

As x2 > x1, then let x2 = x1 + x1, where x1, represents a small change in x1. Therefore, x2 = x1 + x1 and f(x2) = f(x1 + x1).

Substituting for x2 and f(x2) in equation (3.2), we have y f(x1 + x1) f(x1) = x (x1 + x1) x1 = f(x1 + x1) f(x1) x1 (3.3)

Equation (3.3) represents the slope of the straight line AB, rather than of the curve AB. If we keep on making x1 smaller, we approach a point such as A and obtain a line that touches the curve only at the point A. This line is the tangent to the curve at the point A (tangent at a point is defined as the line that touches the curve only at that point and does not cross the curve at that point). Now when x1 is very small, point B will be extremely close to A. In mathematics, this is known as taking the limit of the ratio y as x1 0. Hence from equation (3.3) we have, x

slope of the curve at point A = Lt. x 0

f(x1 + x1) f(x1) . x1

UNIT 1 79
Basic mathematics for management

In general, the slope of the curve at any point A(x, y) is defined as: dy y f(x1 + x1) f(x1) = Lt. = Lt. . x1 dx x 0 x x 0 Hence, we can say that the derivative of a function is the generalised expression for the slope of a function. Furthermore, if we can calculate the derivative at any point on a curve, this means that we know the value of the slope at that point. Another interpretation of the derivative dy is that it measures the rate of change of the dx

[ ] [

variable y with respect to the variable x. At any point where the limit of (3.3) does exist, the function y = f(x) is said to have a derivative or to be differentiable and dy is said to be the first derivative or the dx dy are dx

derivative of y = f(x). The process of obtaining the first derivative of a function is referred to as differentiation. Various types of notations, in addition to

used to denote the first derivative of y = f(x) with respect to x. The most common of these are f (x); y; d (y); Dx(y) dx

3.4 Rules of differentiation


Some of the most commonly used rules of differentiation are as follows:

Polynomial functions
1. Derivative of a constant Let y = K, where K is a constant, then dy = 0. dx

Example 5
a) If y = 10, then dy = 0. dx

b) If y = 0, then

dy = 0. dx

That is, the derivative of a constant is always zero.

80 WAWASAN OPEN UNIVERSITY


BBM 511 Quantitative Techniques

2. Derivative of a power fuction Let y = kxn, where the coefficient k and exponenet n are constants, then dy d d = (kxn) = k (xn) dx dx dx = k . nxn1. That is, (1) the derivative of the product of a constant and a differentiable function is the product of the constant and the derivative of the function, and (2) the derivative of the power function xn equals the product of the exponent n and the variable x raised a power of one less than the exponent, i.e., (n 1).

Example 6
a) If y = 10x, then dy = 10.1x11 = 10x = 10. dx dy = 10.4x41 = 40x3. dx

b) If y = 10x4, then

3. Derivative of a sum (or a difference) of two (or more) functions Let y = u v, where u = f(x) and v = g(x) are differentiable functions of x, then dy du dv = . dx dx dx That is, derivative of the sum (or difference) of a finite number of differentiable functions equals the sum (or difference) of derivatives of the individual functions.

Example 7
a) If u = 10x4, v = 5x2 and y = u + v, then dy du dv d d = + = (10x4) + (5x2) dx dx dx dx dx = 40x3 10x.

UNIT 1 81
Basic mathematics for management

b) If u =

5 , v = 4x + 9x2 and y = u v, then x2 dy du dv d d = = (5x2) (4x + 9x2) dx dx dx dx dx = 10x3 (4 + 18x) = 10 4 18x. x3

Algebraic functions
1. Derivative of a product of two functions Let y = u . v, where u = f(x) and v = g(x) are differentiable functions of x, then dy dv du =u. +v. dx dx dx

That is, the derivative of the product of two functions equals this sum: (first funtion) (derivative of second function) + (second function) (derivative of first function).

Example 8
If u = x3 + 5, v = x2 and y = u . v then dy d d = (x3 + 5) (x2) + x2 (x3 + 5) dx dx dx = (x3 + 5)(2x) + x2 (3x2 + 0) = 2x4 + 10x + 3x4 = 5x4 + 10x

82 WAWASAN OPEN UNIVERSITY


BBM 511 Quantitative Techniques

2. Derivative of a quotient of two functions u Let y = , v 0, where u = f(x) and v = g(x) are differentiable functions v of x, then du dv dy v . dx u . dx = . dx v2 That is, the derivative of the quotient of two funtions equals: (the denominator) (derivative of the numerator) (the numerator) (derivative of the denominator), then this difference is divided by the square of the denominator.

Example 9
u If u = x2 + 5x + 1, v = x + 2 and y = , then v dy = dx = d d (x + 2) dx (x 2 + 5x + 1) (x 2 + 5x + 1) dx (x + 2) (x + 2)2 (x + 2)(2x + 5 + 0) (x2 + 5x + 1)(1 + 0) (x + 2)2 x2 + 4x + 9 . (x + 2)2

3. Derivative of the nth power of a function Let y = un where u = f(x) is a differentiable function of x and n is any number (positive or negative, integer or non-integer), then dy d = n . un1 (u) dx dx That is, the derivative of such function equals the product of power n, the (n 1)th power of the function and derivative of the function.

Special case: If u = f(x) = x, then un = xn and dy = n . x n1 for any real number n. dx

UNIT 1 83
Basic mathematics for management

Example 10
Let u = x2 + 2x, y = u 3 = (x2 + 2x) 3, then
1 1 dy 1 d 2 = (x2 + 2x) 3 (x + 2x) dx 3 dx 4 1 = (x2 + 2x) 3 (2x + 2) 3 1 1

(2x + 2) 3(x2 + 2x)3


4

4. Derivative of a function of a function Let y = f(u) and u = g(x), then dy dy du = . dx du dx

Example 11
Let y = u2 and u = 2x + 3, therefore dy = 2u = 2(2x + 3) = 4x + 6 du du = 2 + 0 = 2. dx Hence, dy dy du = . dx du dx = (4x + 6) . 2 = 8x + 12

Logarithmic functions Let y = loga u, where u = f(x) is a differentiable function. Then dy 1 d = log a. (u) e dx dx u Note: log = loge .

84 WAWASAN OPEN UNIVERSITY


BBM 511 Quantitative Techniques

Example 12
a) Let y = (log x2)2, then dy d = 2 . (log x2) (log x2) dx dx = 2(log x2) 2 log x2 x

1 . 2x x2

b) Let y = log

( )

x dy , then = x+1 dx

( ) ( ) ] ( )[
1 d x . x dx x + 1 x+1 1 x x+1 (x + 1)1 x . 1 (x + 1) 2

( )[

x+1 1 1 = 2 x(x + 1) x (x + 1) dy 1 du = . . dx u dx

Special case: If a = e, then y = logeu and

Exponential functions 1. Let y = au where u = f(x) is a differentiable function of x, then dy u d = a loge a (u) dx dx Special case: If a = e, then y = eu and dy u du =e . dx dx

Example 13
a) Let y = 2x
2 +1

then

dy d 2 = 2x +1 . loge 2 . (x2 + 1) dx dx = 2x
2 +1

. log e 2 . (2x + 0)
2 +1

= 2x . 2x

loge 2

UNIT 1 85
Basic mathematics for management

d x d x ex dy x . dx (e ) e dx (x) b) y = , then = x dx x2 = x . ex ex . 1 ex(x 1) = x2 x2

2. Let y = uv or loge y = v loge u, where u = f(x) and v = g(x) are differentiable functions of x, then d d 1 dy d d (loge y) = (v . loge u) = . = v (loge u) + loge u (v) dx dx y dx dx dx =v

or

v du dv dy + loge u =y . dx u dx dx = uv

( ) [ [

1 du dv . + loge u u dx dx

v du dv . + loge u u dx dx

( ) ] ]

= v . uv1

du dv + uv loge u dx dx

It may be noted that rule (1) is a special case of rule (2) where u = a.

Example 14
Let y = xx +1, then taking log on both sides, we have loge y = (x2 + 1)loge x. Differentiating both sides with respect to x, we have 1 dy 1 = (2x + 0)loge x + (x2 + 1) y dx x = 2x loge x + x + 1 x 1 x
2

()

= x (2 loge x + 1) +

86 WAWASAN OPEN UNIVERSITY


BBM 511 Quantitative Techniques

Activity E
Find the derivatives for the following functions: 1. y = 5x 2. y = 5x2 + 3x + 2 x+1 x1

3. y =

4. y = (x + 2)2 . (x2 1)4 5. y = (log10 x2)3 6. y = loge (3x2 5x) 7. y = 5x 8. y = x4 . elogx 9. y = xe


x

3.5 Applications of the derivative


In economics, variation of one quantity y with respect to another quantity x is usually described in terms of two concepts, as follows: 1. Average concept. 2. Marginal concept. The average concept expresses the variation of y over a whole range of values of x. It is usually measured from zero to a certain selected value, say from 5 to 10 whereas marginal concept is concerned with the instantaneous rate of change in the dependent variable y for every small variation of x from a given value of x. Therefore a marginal concept is precise only when variation in x is made smaller i.e., considering its limiting value only.

Hence,Lt. x 0

( )

y is interpreted as the marginal value of y. x

UNIT 1 87
Basic mathematics for management

Few applications of the derivative are discussed below:

Average and marginal cost


Suppose the total cost y of producing and marketing x units of an item is represented by the function y = f(x). Then the average cost which represents the cost per unit is given by y f(x) Average cost (AC) = or . x x

Now, if the output is increased from x to x + x, and the corresponding total cost becomes y + y, then the average increase in cost per unit output is given by the ratio y and the marginal cost is defined as: x Marginal cost (MC) =Lt. x 0

( )

y dy = . x dx

That is, the marginal cost is the first derivative of the total cost y with respect to output x and is the rate of increase in total cost with increase in output.

Example 15 The total cost, C(x) associated with producing and marketing x units of an item is given by C(x) = 0.005x3 0.02x2 30x + 3000 (in RM). Find: 1. Total cost when output is 4 units. 2. Average cost when output is 10 units. 3. Marginal cost when output is 3 units.

Solution: 1. Given that C(x) = 0.005x3 0.02x2 30x + 3000. For x = 4 units, the total cost C(x) becomes C(4) = 0.005(4)3 0.02(4)2 30(4) + 3000 = 0.32 0.32 120 + 3000 = RM2880.

88 WAWASAN OPEN UNIVERSITY


BBM 511 Quantitative Techniques

2. Average cost (AC) =

C(x) x 0.005x3 0.02x2 30x + 3000 x 3000 x

= 0.005x2 0.02x 30 +

Average cost at x = 10 units becomes AC = 0.005(10)2 0.02(10) 30 + 3000 10

= 0.5 0.2 30 + 300 = RM270.30 per unit. dC . dx

3. Marginal cost at x is given by

Therefore differentiating both sides of C(x) with respect to x, we have d (C(x)) = 0.005 3x2 0.02 2x 30. Marginal cost at x = 3 becomes dx dC = 0.015(3)2 0.04(3) 30 dxx=3 = 0.135 0.120 30 = RM30.015 per unit.

Activity F
The sales S for a product with price x is given by S = 2000e0.5x. Find: 1. Total sales revenue, S = 2000e
0.5x

at x = 2.

2. Marginal revenue,

dS at x = 2. dx

UNIT 1 89
Basic mathematics for management

Total revenue, marginal revenue and average revenue


Let p be the price per unit and q is the number of units of an item sold. Then the total revenue (R) is given by R=p.q The demand function is p = f(q). therefore R becomes R = q . f(q) Now average revenue (AR) or revenue per unit which represents the price per unit is given by AR = R p.q = = p (price). q q

This shows that the average revenue and price are identical. Since total revenue is given by R = p . q, therefore marginal revenue (MR) is defined as: MR = dR dp = p + q. dq dq

=p1+

q dp . . p dq

Example 16 The demand for a certain product is represented by the equation: p = 20 + 5q q2 where q is the number of units demanded and p is the price per unit (in RM). Find the marginal revenue function. What is the marginal revenue at q = 2?

Solution: The total revenue is given by Revenue, R = (demand)(price) = q(20 + 5q q2) = 20q + 5q2 q3. Marginal revenue (MR) = d (20q + 5q2 q3) dq

= 20 + 10q 3q2.

90 WAWASAN OPEN UNIVERSITY


BBM 511 Quantitative Techniques

The marginal revenue (MR) at q = 2 is given by MR = dR = 20 + 10q 3q2 dq

= 20 + 10(2) 3(2)2 = 20 + 20 12 = 28. Hence, the marginal revenue when two units are demanded is RM28.

Activity G
The demand for a certain product is represented by the equation p = 300 6q where p is the price per unit and q is the number of units demanded. Find the revenue function. What is the slope of the revenue function? At what price is the marginal revenue zero?

Elasticity
The elasticity of a function y = f(x) at a point x is defined as the ratio of the rate of proportional change in y per unit of proportional change in x. That is, Ey y x dy = = . Ex dx y dx
x dy

The elasticity of a function is independent of the units in which the variables are measured because its definition is in terms of proportional changes. Notations usually used to denote elasticity are ey, or y or yx. The above definition can also be expressed as:
dy dy y dx x dx y x

ey =

Marginal function . Average function

The crucial value of ey = 1. However the sign of ey depends upon the sign of

dy . It dx

may be positive, negative or zero. Apart from the sign, we are also concerned about the absolute value |ey| of ey.

UNIT 1 91
Basic mathematics for management

1. Price elasticity of supply Let g be the supply and p be the price and the function is expressed as q = f(p). Then the formula for elasticity of supply is same as that of ey. That is es = p dq . q dp

The sign of es will also be positive because the slope of the supply curve is positive.

2. Price elasticity of demand The price elasticity of demand at price b is defined as: ed = p q Lt. p 0 q p

{ }
dq

p dq p 1 = . = . q dp q dp dq is dp

The sign of ed is negative, because in general the slope of demand negative.

3. Marginal revenue and elasticity of demand You know that the total revenue (R) is given by R=p.q where p is the price and q is the quantity sold. Also the average revenue (AR) and marginal revenue (MR) are defined as: Average revenue (AR) = R p.q = =p q q dR d = = (p .q) dq dq dp dq

Marginal revenue (MR) =

=p.1+q

92 WAWASAN OPEN UNIVERSITY


BBM 511 Quantitative Techniques

=p 1+

=p 1+

( (

q dp . p dq 1 |ed|

) )

= AR 1 + p dq . . q dp

1 |ed|

since |ed| =

From this definition of MR, it follows that: 1. If |ed| = 1, then AR = 0 and hence MR = 0, i.e., total revenue remains constant with a fall in price. 2. If |ed| > 1, then AR = 0 and hence MR > 0, i.e., total revenue increases with an increase in demand or with a fall in price. 3. If |ed| > 1, then AR = 0 and hence MR < 0, i.e., total revenue decreases with an increase in demand or with a fall in price.

Example 17 Suppose the price p and quantity q of a commodity are related by the equation q = 30 4p p2 Find:
dq

1. Elasticity of demand, eq is defined as

q dp p

at p = 2.

2. Marginal revenue (MR) defined as

dR , where R = p . q. dq

UNIT 1 93
Basic mathematics for management

Solution: 1. Elasticity of demand, eq is defined as:


dq q dp p

eq =

p dq = . q dp

p d = . (30 4p p2) q dp = p . (4 2p) (30 4p p2)

4p + 2p2 . 30 4p p2 4 2 + 2(2)2 16 8 = = . 2 30 4 2 (2) 18 9

Thus eq =

2. Marginal revenue (MR) is defined as: MR = dR dR dp = . dq dp dq dR . dp 1


dq dp

d(p . q) . dp

1
dq dp

d [p . (30 4p p2)] . dp

1
d dp

(30 4p p2)

= [30 8p 3p2] . 30 8p 3p2 . 4 2p

1 (0 4 2p)

94 WAWASAN OPEN UNIVERSITY


BBM 511 Quantitative Techniques

Activity H
The demand q (in kg) for a commodity with its price p (in RM) is given by

p = 108

( )

3 . 5q

Find the elasticity of demand when the price is RM12.

3.6 Concept of maxima and minima with managerial applications


The objective of studying differential calculus is to be able to solve optimisation problems in which the decision maker seeks either to maximise or minimise the given objective function (or goal) under certain limitations (or constraints) on available resources. In this section, unconstrained optimisation problems involving single independent variable are presented.

Conditions for maxima and minima


The necessary condition Consider the function y = f(x) given in Figure 9(a). At the point A which is the lowest point of the curve, the tangent is neither inclined to the right nor to the left. But the tangent is parallel to the x-axis and its slope is zero, i.e., m = tan = 0 because the slope of a horizontal line is equal to zero. The slope is measured by the first derivative, therefore the derivative at point A must be equal to zero. y

Negative slope C A

Positive slope slope = 0

0
Figure 9(a)

x=ah

x=a

x=a+h

UNIT 1 95
Basic mathematics for management

From Figure 9(a), it is clear that the value of the function y = f(x) decreases as x increases up to A, i.e., increases from x = a h to x = a and then increases as x increases up to B, i.e., increases from x = a to x = a + h. Thus dy will be negative up dx

to A, becomes zero at A and will be positive after crossing A. This shows that if the function f(x) is minimum at point A, then its first derivative at point A is equal to zero, but the converse is not true. That is, dy = 0 at point A. dx This minimum value of the function y = f(x) at x = a is called local (or relative) minimum value because the value y = f(a) is less than any other value of f(x) for x in an interval around a. The word local (or relative) has been used to define this minimum value of f(x) because it has been obtained with reference to a small interval containing the point. From Figure 9(b), it is clear that the function f(x) reaches a maximum at the point D. It can also be verified that function f(x) increases as x increases up to D and then decreases after crossing D. Thus dy will be positive up to D, becomes zero at D dx

and will be negative after crossing D. This also shows that if the function f(x) is maximum at point D, then its first derivative at that point is zero but the converse is not true. That is, dy = 0 at point D. dx y D E Positive slope F Negative slope

slope = 0

0
Figure 9(b)

x=ah

x=a

x=a+h

96 WAWASAN OPEN UNIVERSITY


BBM 511 Quantitative Techniques

This maximum value of the function f(x) at x = a is called a local (or relative) maximum because y = f(a) is greater than any value of f(x) for x in an interval around a. Hence, the condition that the first derivative is equal to zero at the maxima (plural of maximum) or minima (plural of minimum) is a necessary condition but not a sufficient one because it does not help us to locate the absolute (or global) maximum or minimum. The absolute maximum (or minimum) means the maximum (or minimum) value of f(x) amongst all given maximum (or minimum) values in the specified interval for x.

The sufficient condition


The function y = f(x) whose graph is given in Figure 9(c) has four maxima and four minima in the entire range from x = b to x = c. y

Relative maximum

Absolute maximum bxc H G f(c)

D B Absolute minimum bxc A f(b) C Relative minimum E

Figure 9(c)

The slope of the curve at the points A to H is zero. Such points for which

dy = 0 are dx

called the stationary points or extreme points or critical points of the function y = f(x). The function has maxima at the points B, D, F, H and minima at the points, A, C, E, G. The absolute (or global) maximum occurs at the point F and the absolute (or global) minimum occur at the point A. However these values of a function in an interval may occur at an end point of the interval rather than at a relative minimum or maximum value. Let us now examine the sign of minima. dy in the neighbourhood points of maxima and dx

UNIT 1 97
Basic mathematics for management

1. The sign of

dy changes from positive to negative as x passes through the dx dy points of maxima. If you consider as a function of x, then you will find dx dy is negative. In other words, dx

that it is a decreasing function as it passes through the points of maxima, i.e., rate of change of d dy <0 dx dx or d2y <0 dx2 at a point where f(x) is maximum. dy changes from negative to positive as x passes through the dx dy points of minima, and hence is an increasing function, i.e., rate of dx dy change of is positive. In other words dx d dy >0 dx dx or d2y >0 dx2 at the point where f(x) is a minimum. d2y = 0. dx2

( )

2. The sign of

( )

However, at certain points, you may find

Such points are called point of inflexion. In such cases, the points are neither maximum nor minimum.

98 WAWASAN OPEN UNIVERSITY


BBM 511 Quantitative Techniques

Summary of the results Maximum Necessary condition Sufficient condition dy =0 dx dy d2y = 0, 2 < 0 dx dx Minimum dy =0 dx dy d2y = 0, 2 > 0 dx dx

Summary of the procedure


1. Take the first derivative of the given function. 2. Set the derivative equal to zero and solve the values of the independent variable at which the function is either maximum or minimum. 3. Take the second derivative of the function. 4. Evaluate the second derivative at the points obtained in step 2. 5. If the second derivative is positive, then f(x) is minimum at the given point. Otherwise the point is a maximum point.

Example 18 Suppose a manufacturer can sell x items per week at a price, p = 20 0.001x (in RM) each when it costs y = 5x + 2000 (in RM) to produce x items. Determine the number of items he should produce per week for maximum profit.

Solution: The cost of producing x items = 5x + 2000. The price of one item = 20 0.001x. Therefore, the selling price of x items = x(20 0.001x). Let z be the profit function given by: z = Revenue Cost = (20x 0.001x2) (5x + 2000) = 0.001x2 + 15x 2000 and dz = 0.002x + 15. dx

UNIT 1 99
Basic mathematics for management

For maximum profit, dz = 0.002x + 15 = 0 dx or 0.002x = 15 x= 15 = 7500 0.002

d2z d dz = dx2 dx dx =

()

d (0.002x + 15) = 0.002(ve). dx

So profit is maximum when 7500 items are produced and sold.

Activity I The cost of fuel for running a train is proportional to the square of the speed generated in kilometres per hour, and costs RM75 per hour, at 17 kilometres per hour. What is the most economical speed, if the fixed charges are RM400 per km per hour? _____________________________________________________________________ _____________________________________________________________________ _____________________________________________________________________ _____________________________________________________________________ _____________________________________________________________________

Summary
The objective of this section is to provide you with some exposure to differential calculus. Differential calculus is useful to solve optimisation problems, problems in which the aim is either to maximise or minimise a given objective function. Because of this reason, it has found wide applications in this field. Applications of the derivative in both micro economics theory (cost, revenue, elasticity) and macro economic theory (income, consumption, savings) are good examples of its application in business. The section begins with a discussion on the concepts of limit and continuity and then attention is directed to defining the slope of a linear function and proceeds with a discussion that extends this to include the slope of a non-linear function. This is followed by the definition of the term derivative and rules for obtaining the derivatives of the more commonly encountered functional forms. The term derivative is a generalised expression for measuring the rate of change

100 WAWASAN OPEN UNIVERSITY


BBM 511 Quantitative Techniques

or slope of a function. Through several examples, the concepts of average cost, marginal cost, total revenue, marginal revenue, average revenue and elasticity are demonstrated by using the derivative first. The procedures for determining local maxima and minima for the given function are demonstrated through an example and graph. A step by step procedure for finding maximum and minimum of a function is outlined. Each subsection in this section is followed by an exercise for students practice.

Keywords
Classical optimisation: Locating the maximum and/or minimum value(s) of a function through the application of differential calculus.

Continuity: A function is said to be continuous at a point x = a if (1) f(a) exists (2) xLt. f(x) exists and (3) xLt. f(x) = f(a). a a

Critical point: Any point that satisfies the necessary condition, may be maxima, minima or points of inflection.

dy = 0. These points dx

Derivative: A function that expresses the slope of another function at every point.

Differential calculus: It is concerned with determining the rate of change of a given function due to one unit of change in one of the independent variables.

Integral calculus: It is concerned with the inverse problem of finding a function when its rate of change is given.

Limit: The method of knowing the behaviour of a function y = f(x) as the independent variable x approaches a particular value. Local maximum: A point on a curve that is higher than the points on both sides of itself. A point where dy d2y = 0 and 2 < 0. dx dx

Local minimum: A point on a curve that is lower than the points on both sides of itself. A point where dy d2y = 0 and 2 > 0. dx dx

UNIT 1 101
Basic mathematics for management

Point of inflection: A point on a curve at which the but d2y = 0. dx2

dy may or may not be zero dx

Slope: The rate of change in the dependent variable (y) for one unit of change in the independent variable (x).

Tangent: A straight line that touches a non-linear function at only one point, not cutting through the curve at the point. The slope of the tangent is used as a measure of the slope of the curve at that point.

Further readings
1. Tan, S T (2007) Applied Mathematics for the Managerial, Life and Social Sciences, 4th edn, Belmont, CA: Thomson Brooks/Cole. 2. Haeussler, E F, Paul, R S and Wood, R J (2005) Introductory Mathematical Analysis for Business, Economics and the Life and Social Sciences, 11th edn, Upper Saddle River, NJ: Pearson Prentice Hall. 3. Lial, M L, Hungerford, T W and Holcomb, J P (2007) Mathematics with Applications In the Management, Natural and Social Sciences, 9th edn, Boston: Pearson Addison Wesley.

102 WAWASAN OPEN UNIVERSITY


BBM 511 Quantitative Techniques

Suggested answers to activities

Activity A
1. a) nLt. 1+

( ) ( )

1 1 =1+ =1 n

2 n(1 n ) n2 = Lt. b) nLt. n n + 1 1 n(1 + n )

= nLt.

( ) ( )
1
2 n

1+1 n

=1

2. S = 2000 + 4000(1 e0.01x)


n

Lt. S = 2000 + 4000(1 0) = 6000

When there is infinite advertising expenditure, the sales of the product will maintain at 6000 (in RM 000).

Activity B
At the point x = 200 RHL = x Lt. C(x) = 15 200 = 3000 200

LHL =x Lt. C(x) = 13 200 = 2600 200

Hence RHL LHL, x = 200 is a point of discontinuity.

At the point x = 400 RHL = x Lt. C(x) = 13 400 = 5200 200

LHL = x Lt. C(x) = 10 400 = 4000 200

Since RHL LHL, x = 400 is another point of discontinuity.

UNIT 1 103
Basic mathematics for management

Activity C
Let y = salary of the salesman, x is the number of items sold. The functional relationship of y and x is given by: y = 500 + 2x. The slope of the line is 2.

Activity D
When x = 8, y = 4(8) + 200 = 57. 8

When x = 9, y = 4(9) + 200 = 58.22. 9

Hence y 58.22 57 = = 1.22. x 91 When x = 20, y = 4(20) + 200 = 90. 20

When x = 21, y = 4(21) + 200 = 93.52. 21

Hence y 93.52 90 = = 3.52. x 21 20

The increase of stock cost will be higher as the order size is higher.

104 WAWASAN OPEN UNIVERSITY


BBM 511 Quantitative Techniques

Activity E
1. y = 5x dy =5 dx

2.

y = 5x2 + 3x + 2 dy = 10x + 3 dx x+1 x1


d d

3.

y=

dy (x 1) dx (x + 1) (x + 1) dx (x 1) = dx (x 1)2 = (x 1) (x + 1) (x 1)2 = 2 (x 1)2

4.

y = (x + 2)2(x2 1)4 dy d d = (x2 1)4 (x + 2)2 + (x + 2)2 (x2 1)4 dx dx dx = 2(x2 1)4(x + 2) + (x + 2)2(4)(x2 1)5(2x) = 2(x2 1)4(x + 2)8x(x + 2)2(x2 1)5 = 2(x2 1)5(x + 2)[x2 1 4x(x + 2)] = 2(x + 2) [3x2 8x 1] (x2 1)5

UNIT 1 105
Basic mathematics for management

5.

y = (log10 x2)3 dy d = 3(log10 x2)2 (log10 x2) dx dx = 3(log10 x2)2

1 (2x) x 1n10
2

6 (log10 x2)2 x 1n10

6.

y = log e (3x2 5x) dy 1 d = 2 (3x2 5x) dx 3x 5x dx = 6x 5 3x2 5x

7.

y = 5x dy d = 5x log e 5 (x) dx dx = 5x log e 5

8.

y = x4elog x dy log x d 4 d log x =e (x ) + x4 (e ) dx dx dx = 4x3 elog x + x4 elog x = 4x3 elog x x3 elog x = 3x3 elog x

()
1 x

106 WAWASAN OPEN UNIVERSITY


BBM 511 Quantitative Techniques

9.

y = xe

log e y = log e xe

= ex log e x 1 dy x d d = e (log e x) + log e x (ex ) y dx dx dx = ex

= ex

1 dy + log e x = yex dx x = xe ex
x

() ( ( (

1 + ex log e x x 1 + log e x x

) ) )

1 + log e x x

Activity F
1. When x = 2, S = 2000e0.5(2) = 2000e1 = 735.7589. dS dS = 1000e0.5x. At x = 2, = 1000e0.5(2) = 1000e1 = 367.879. dt dt

2.

Activity G
p = 300 6q R=p.q = (300 3q)q = 300q 3q2 dR = 300 12q dq The slope of the revenue function = 3 12q.

UNIT 1 107
Basic mathematics for management

When the slope of the revenue function = 0 300 12q = 0 q= 300 = 25 12

p = 300 6(25) = 150.

Activity H
p = 108 3 5q

q=

3 5(108 p) 3 1 = 480 160

When p = 12, q = dp 3 = 2 dq 5q =

1 5 160

()

= 15360 p dq p = q dp q 12

eq =

()
1
dp dq

( )( )
1 160

1 15360

= 0.125.

108 WAWASAN OPEN UNIVERSITY


BBM 511 Quantitative Techniques

Activity I
Let c = cost per hour for the train and v = speed of the train. c v2 c = kv2, where k is a constant. Given c = 75, v = 17. Then k = 75 2 v . 289 75 75 = . 2 17 289

Thus, c =

The revenue function per hour, r = 400v. The profit function p = revenue cost = 400v dp 75 150 = 400 2 v = 400 v. dv 289 289 dp 400(289) = 0 v = = 770.67. dv 150 75 2 v . 289

( )

To maximise the profit,

Since hour.

d2p 150 = < 0, the maximum profit is obtained when v = 770.67 km per dv2 289

UNIT 1 109
Basic mathematics for management

4 Matrix Algebra and its Applications


Objectives
By the end of this section, you should be able to: 1. Explain the basic concepts of a matrix. 2. Apply methods of representing large quantities of data in matrix form. 3. Perform various operations concerning matrices. 4. Find the solution methods of simultaneous linear equations. 5. Discuss applications of matrix algebra in various decision models.

Introduction
Matrices have proven their usefulness in quantitative analysis of managerial decisions in several disciplines like marketing, finance, production, personnel, economics, etc. Many quantitative methods such as linear programming, game theory, Markov models, input-output models and some statistical models have matrix algebra as their underlying theoretical base. All these models are built by establishing a system of linear equations which represent the problems to be solved. The simultaneous linear equations involving more than three variables cannot be solved by using ordinary algebra. Real-world business problems may involve more than three variables, and in such cases matrices are used to represent a complex system of equations and large quantities of data in a compact form. Once the system of equations is represented in matrix form, they can be solved easily and quickly by using a computer. The limitation of matrix algebra is that it is applicable only in those cases where assumption of linearity can be made. The main objective of this section is to provide (1) some basic theoretical matrix operations, namely addition, subtraction, and multiplication (2) a procedure for solving a system of linear simultaneous equations, and (3) a few applications of matrix algebra.

110 WAWASAN OPEN UNIVERSITY


BBM 511 Quantitative Techniques

4.1 Matrices: Definition and notations


A matrix is a rectangular array of ordered numbers. The term ordered implies that the position of each number is significant and must be determined carefully to represent the information contained in the problem. These numbers (also called elements of the matrix) are arranged in rows and columns of the rectangular array and are enclosed by either square brackets [ ], parentheses ( ) or by a pair of double vertical lines || ||. A matrix consisting of m rows and n columns is written in the following form. a column a11 a21 . . am1 am2 amn a12 a22 a1n a2n a row

where a11, a12, denote the numbers (or elements) of the matrix. The dimension (or order) of the matrix is determined by the number of rows and columns. Here, in the given matrix, there are m rows and n columns. Therefore, it is of the dimension m n (read as m by n). In the dimension of the given matrix, the number of rows is always specified first and followed by the number of columns. Boldface capital letters such as A, B, C ... are used to denote the entire matrix. The matrix is also sometimes represented as A = [aij]mxn where aij denotes the element in the ith row and the jth element of A. Some examples of the matrices are 1 1 A= ; 2 3
22

[ ]

1 1 2 B= ; 2 4 3
23

5 5 10 C = 6 2 10 2 1 2

[ ]

33

The matrix A is a 2 2 matrix because it has 2 rows and 2 columns. Similarly, the matrix B is a 2 3 matrix while matrix C is a 3 3 matrix.

UNIT 1 111
Basic mathematics for management

Activity A
Tick the correct alternative indicating the dimension of the matrix

[ ]
2 6 3 3 8 5 4 9 7 A) 3 3 B) 4 3

33

C) None of these.

4.2 Some special matrices


Square matrix
A matrix in which the number of rows equals the number of columns is called a square matrix. For example

[ ]
2 6 3 3 8 5 4 9 7

33

is a square matrix of dimension 3. The elements 2, 8 and 7 in this matrix are called the diagonal elements and the diagonal is called the principal diagonal.

Diagonal matrix
A square matrix, in which all non-diagonal elements are zero whereas diagonal elements are non-zero, is called a diagonal matrix. For example

[ ]
2 0 0 0 5 0 0 0 1

33

is a diagonal matrix of dimension 3.

112 WAWASAN OPEN UNIVERSITY


BBM 511 Quantitative Techniques

Scalar matrix
A diagonal matrix in which all diagonal elements are equal is called a scalar matrix. For example

[ ]
k 0 0 0 0 k 0 0 k

33

is a scalar matrix, where k is a real (or complex) number.

Identity (or unit) matrix


A scalar matrix in which all diagonal elements are equal to one, is called an identity (or unit) matrix and is denoted by I. Following are two different identity matrices 1 I2 = 0

[ ]

0 ; 1
22

1 0 0 I3 = 0 1 0 0 0 1

[ ]

33

An identity matrix of dimension n is denoted by In. It has n elements in its diagonal, each equal to 1 and other elements are zero.

The zero (or null) matrix


A matrix is said to be the zero matrix if every element of it is zero. It is denoted as 0. Following are three different zero matrices:

[ ]

0 0 ; 0 0
22

0 0 0 ; 0 0 0
23

[ ]
0 0 0 0 0 0

32

4.3 Matrix representation of data


Before discussing the operations on matrices, it is necessary for you to know a few situations in which data can be represented in matrix form.

UNIT 1 113
Basic mathematics for management

Transportation problem
The unit cost of transportation of an item from each of the two factories to each of the three warehouses can be represented in a matrix as shown below: Warehouses F1 F2

Factory

W1 20 25

W2 15 20

W3 30 15

Similarly, we can also construct a time matrix [tij], where tij = time of transportation of an item from factory i to warehouse j. Note that the time of transportation is independent of the amount shipped.

Distance matrix
The distance (in km) between the given number of cities can be represented as a matrix as shown below: City A City B C D

A 1,470 2,158 1,732

B 1,470 1,853 2,365

C 2,158 1,853 1,635

D 1,732 2,385 1,635

Diet matrix
The vitamin content of two types of food and two types of vitamins can be represented in a matrix as shown below: Vitamins F1 F2

Food

A 150 170

B 120 100

114 WAWASAN OPEN UNIVERSITY


BBM 511 Quantitative Techniques

Assignment matrix
The time required to perform three jobs by three workers can be represented in a matrix as shown below: Job W1 Worker W2 W3

J1 5 4 2

J2 3 5 4

J3 2 3 6

Pay-off matrix
Suppose two players A and B play a coin tossing game. If the outcome of (H, H) or (T, T) occurs, then player B loses RM10 to player A, otherwise player B gains RM10 from player A as shown in the matrix:

Player B H Player A H T T 10 10

10 10

The minus sign with the pay-off means that player A pays to player B.

Brand switching matrix


The proportion of users in the population surveyed switching from brand i to brand j of an item in a period can be represented as a matrix: To Brand 1 Brand 1 From Brand 2 Brand 3

Brand 2 0.6 0.3 0.5

Brand 3 0.1 0.1 0.3

0.3 0.6 0.2

Here the sum of the elements of each row is 1 because these are the proportions.

UNIT 1 115
Basic mathematics for management

4.4 Operations on matrices


Addition (or subtraction) of matrices
The addition (or subtraction) of two or more matrices is possible only if these matrices have the same dimensions, i.e., matrices must have the same number of rows and the same number of columns. The sum (or difference) of matrices is obtained by adding (or subtracting) the corresponding elements of the given matrices. For example, if

A=

[ ]
1 2 3 4

and

B=

22

[ ]
1 7 0 8 0 2

22

then A+B=

[ [

11 2+0

3+7 4+8

22

[ ]
10 12 = 2 2

22

AB=

1 (1) 20

37 48

[ ]
4 4

Note that A B B A.

Example 1 A company produces three types of products A, B and C. The total annual sales (in 000 units) of these products for the years 2005 and 2006 in the four regions are given below. For the year 2005: Region Product A B C Eastern 15 5 8 Western 8 24 4 Southern 6 7 31 Northern 12 8 6

116 WAWASAN OPEN UNIVERSITY


BBM 511 Quantitative Techniques

For the year 2006: Region Product A B C Eastern 17 5 13 Western 10 22 6 Southern 5 11 39 Northern 7 4 5

Find the total sales of three products for two years.

Solution: The total sales of three products for two years can be obtained by adding the sales of two years as shown below: Region Product A B C Eastern 15 + 17 = 32 5 + 5 = 10 8 + 13 = 21 Western 8 + 10 = 18 24 + 22 = 46 4 + 6 = 10 Southern 5 + 5 = 10 7 + 11 = 18 31 + 39 = 70 Northern 12 + 7 = 19 8 + 4 = 12 5 + 6 = 11

Properties of matrix addition


If A, B and C are any three matrices of same dimension, then 1. Matrix addition is commutative, i.e., A + B = B + A.

2. Matrix addition is associative, i.e., (A + B) + C = A + (B + C).

3. For any matrix A of dimension m n, there is a zero matrix of the same dimension such that A + 0 = 0 + A = A. This shows that zero matrix is the additive identity.

UNIT 1 117
Basic mathematics for management

4. If for any matrix A of dimension m n, there exists another matrix B of the same dimension such that A+B=B+A=O then B is called the additive inverse (or negative) of A and is denoted by A.

Activity B
If matrices A and B are defined as

A=

0 2

2 3 ; 1 4

B=

7 1

6 4

3 5

then compute: a) A + B b) A B c) B A

Scalar multiplication
If A= [aij] is any matrix of dimension m n and k is any scalar (real number), then the multiplication kA is obtained by simply multiplying each element of A by the scalar k. That is Ak = kA = [kaij]

Example 2 The sales figures in Example 1 are given in thousands of units. If we want to express the sales figures in actual units, then we have to multiply the given matrices by 1000. For illustration, let us consider the data matrix of 1985. That is, if 15 A= 5 8

8 24 4

5 7 31

12 8 6

118 WAWASAN OPEN UNIVERSITY


BBM 511 Quantitative Techniques

then 15 1000 1000 A = 5 1000 8 1000 15,000 = 5,000 8,000

[ [

8 1000 24 1000 4 1000 8,000 24,000 4,000

5 1000 7 1000 31 1000 12,000 8,000 6,000

12 1000 8 1000 6 1000

5,000 7,000 31,000

Properties of scalar multiplication


1. k(A + B) = kA + kB where A and B are two matrices of same dimension and k is a scalar number.

2. (k1 + k2) A = k1A + k2A where A is a matrix and k1 and k2 are two distinct scalar numbers.

Activity C
If two matrices A and B are defined as

A=

0 2

2 3 ; 1 4

B=

7 1

6 4

3 5

then compute 2A + 3B.

Multiplication of matrices
The matrix multiplication consists of the following steps: 1. Check on compatibility: The following dimensional arrangement must hold for compatibility in matrix multiplication: Dimensions: lead matrix lag matrix = product (m p) (p n) = m n In other words, the number of columns in the first matrix must be equal to the number of rows in the second matrix. If this condition does not exist, then the matrices are said to be incompatible and their multiplication is not defined.

UNIT 1 119
Basic mathematics for management

2. The operation of multiplication: For multiplication of two matrices, the following procedure should be adopted: a) The element of a row of the lead matrix A should be multiplied by the corresponding elements of a column of the lag matrix B. b) The product is then added and the location of this resulting element in the new matrix C determines which row from A has to be multiplied with which column from B.

To illustrate this, let us take two matrices A and B as defined below:

A=

2 3

3 5 ; 5 7
23

2 5 B= 3 5 5 7

[ ]

32

then A (2 3) R1 R2 B (3 2) 2 3 5 5 5 7 = C (2 2)

2 3

3 5

5 7

[ ][
C1 C2

= (2)(2) + (3)(3) + (5)(5) (2)(5) + (3)(5) + (5)(7) (3)(2) + (5)(3) + (7)(5) (3)(5) + (5)(5) + (7)(7) = 38 56

60 89

22

Example 3 There are two families A and B. There are 2 men, 3 women and 1 child in family A and 1 man, 1 woman and 2 children in family B. The recommended daily allowance for calories is: man, 2400; woman, 1900; child, 1800, and for proteins: man, 55 gm; woman, 45 gm and child, 33 gm. Represent the above information by matrices. Using matrix multiplication, calculate the total requirement of calories and proteins for each of the two families.

Solution: man Let C = Family and A B 2 1 women 3 1 child 1 2


23

120 WAWASAN OPEN UNIVERSITY


BBM 511 Quantitative Techniques

D = Man Woman Child

Calory 2400 1900 1800

Protein 55 45 33

32

If you look at the dimensions of two matrices C and D you will find that the condition for multiplication is satisfied. Therefore, the total requirement of calories and proteins for each of the two families is determined by multiplying C and D, as shown below: R1 R2

CD

2400 55 2 3 1 R 1C 1 R 1C 2 1900 45 = 1 1 2 R2C1 R2C2 1800 33

][

22

C1 Calory Family A B

C2 Protein 278 166

12300 7900

22

Activity D
1. If two matrices of dimension m n and n p are multiplied, then the resulting matrix is of dimension: A) m n B) n p C) m p D) None of these.

2. If A and B are two non-zero compatible matrices with respect to multiplication, then their product A) is always zero matrix. B) is never a zero matrix. C) may be a zero matrix. D) None of these.

UNIT 1 121
Basic mathematics for management

3. A factory employs 50 skilled workers and 20 unskilled workers. The daily wages to skilled and unskilled workers are RM30 and RM17 respectively. Using matrix notation, find: a) The matrix representing the number of workers. b) The total daily payment made to the workers.

Properties of matrix multiplication


1. Matrix multiplication, in general, is not commutative, i.e., AB BA.

2. Matrix multiplication is associative, i.e., A(BC) = (AB)C where A, B, C are any three matrices of dimension m n, n p and p q respectively.

3. Matrix multiplication is distributive, i.e., A(B + C) = AB + AC where A, B, C are any three m n, n p and n p matrices respectively.

Transpose of matrix
Let A be any matrix. The matrix obtained by interchanging rows and columns of A is called the transpose of A and is denoted by A or At. Thus if A = [aij] is an m n matrix, then At = [aji] will be n m matrix. For example, the transpose of the matrix

A=

2 3 1 2

4 0

23

is 2 1 At = 3 2 4 0

[ ]

32

122 WAWASAN OPEN UNIVERSITY


BBM 511 Quantitative Techniques

Properties of transpose of matrices


1. Transpose of a sum (or difference) of two matrices is the sum (or difference) of the transposes, i.e., (A B)t = At Bt.

2. Transpose of a transpose is the original matrix (At)t = A.

3. Transpose of a product of two matrices is the product of their transposes taken in reverse order (AB)t = Bt At.

Activity E
If two matrices A and B are defined as 2 A= 2

1 2 ; 4 0

2 2 B= 1 4 2 0

[ ]

then verify that (AB)t = Bt At.

4.5 Determinant of a square matrix


The determinant of a square matrix is a scalar (i.e., a number). Determinants are possible only for square matrices. For more clarity, we shall be defining it in stages, starting with square matrix of order 1, then for matrix of order 2, etc. The determinant of a square matrix A is denoted either by |A| or det. A. 1. Determinant of order 1. Let A = (a11) be a matrix of order 1. Then det. A = a11. 2. Determinant of order 2. Let A=

a11 a21

a12 a22

UNIT 1 123
Basic mathematics for management

be a square matrix of order 2, then the determinant of A is defined as det. A =

a11 a12 = a11 a22 a21 a12 a21 a22

For example, det. A =

| |

3 4 = 3 2 1 4 = 2. 1 2

To write the expansion of a determinant to matrices of order 3, 4, ..., let us first define two important terms: Minor: Let A be a square matrix of order m. Then the minor of an element aij is the determinant of the residual matrix (or submatrix) obtained from A by deleting row i and column j containing the element aij. In the matrix A,

a11 a12 a21 a22 a31 a32

a13 a23 a33

]
| |

The minor of the element aij is denoted by Mij. The minor of the element a11 is obtained by deleting the first row and first column containing element a11 and is written as

M11 =

| |

a22 a23 . a32 a33

Similarly, minor of a12 is a21 a23 . a31 a33

M12 =

Cofactor: The cofactor cij of an element aij is defined as cij = (1)i+jMij where Mij is the minor of an element aij.

124 WAWASAN OPEN UNIVERSITY


BBM 511 Quantitative Techniques

Now using the concept of minor and cofactor, you can write the expansion of a determinant of order 3 as shown below:

a11 a21 a31

a12 a22 a32

a13 a23 a33

= a11 C11 + a12 C12 + a13 C13 = a11 (1)1+1 M11 + a12 (1)1+2 M12 + a13(1)1+3 M13. a23 a + a13 21 a33 a31

= a11

a11 a32

a22 a a12 21 a33 a31

| |

| |

a22 a32

= a11(a22 a33 a32 a23) a12(a21 a33 a31 a23) + a13(a21 a32 a31 a22).

The expansion of the given determinant can also be done by choosing the elements of any row and column. In the above example, expansion was done by using the elements of the first row.

Example 4 Find the value of the determinant, 1 det. A = 2 2

18 40 45

72 96 . 75

Solution: If you expand the determinant by using the elements of the first column, then you will get

1 2 2

18 40 45

72 40 96 = 1 45 75

96 18 2 75 45

| |

72 18 72 +2 75 40 96

| |

= 1(3000 4320) 2(1350 3240) + 2(1728 2880) = 1 (1320) 2 (1890) + 2(1152) = 1320 + 3780 2304 = 3624 + 3780 = 156.

UNIT 1 125
Basic mathematics for management

Properties of determinants
The following are useful properties of determinants of any order. These properties are very useful in expanding the determinants. 1. The value of a determinant remains unchanged if rows are changed into columns and columns into rows, i.e., |A| = |At|. 2. If two rows (or columns) of a determinant are interchanged, then the value of the determinant obtained is the negative of the original determinant. 3. If each element in any row or column of a determinant is multiplied by a constant number, say K, then the determinant obtained is K times the original determinant. 4. The value of a determinant in which two rows (or columns) are equal is zero. 5. If any row or column of a determinant is replaced by the sum of the row and a linear combination of other rows (or columns), then the value of the determinant so obtained is equal to the value of the original determinant. 6. The rows (or columns) of a determinant are said to be linearly dependent if |A| = 0, otherwise they are independent.

Example 5 Verify the following result.

| |

1 a a2 1 b b2 = (a b) (b c) (c a). 1 c c2

Applying row operations (Property 5), R2 R2 + (1)R1 R3 R3 + (1)R1

The determinant obtained is:

1 0 0

a a2 2 b a b a2 c a c2 a2

126 WAWASAN OPEN UNIVERSITY


BBM 511 Quantitative Techniques

Expanding the new determinant by the elements of first column, you will get:

ba ca

b2 a2 ba = 2 2 c a ca

||

(b a)(b + a) . (c a)(c + a)

By performing row operations below: R2 1 R (b a) 1 R (c a)

R3

You will have (b a)(c a)

1 1

b+a c+a

= (b a)(c a){(c + a) (b + a)} = (b a)(c a)(c b) = (a b)(b c)(c a)

Activity F
If a + b + c = 0, then verify the following result.

| |

a b c 0 a b = c(2ab c2) b 0 a

4.6 Inverse of a matrix


For a given square matrix A, if another square matrix B of the same order fulfils the condition AB = BA = I then matrix B is called the inverse of A and is denoted by B = A1.

UNIT 1 127
Basic mathematics for management

Before discussing the procedure of finding the inverse of a matrix, it is important to know the following results: 1. The matrix B = A1 is said to be the inverse of matrix A if and only if AA1 = A1 A = I.

2. That is, if the inverse of a square matrix is multiplied by the original matrix, 1 I then the result is an identity matrix. The inverse A1 does not mean or . A A This is simply a notation to denote the inverse of A.

3. Every square matrix may not have an inverse. For example, zero matrix has no inverse because the inverse of square matrix exists only if the value of its determinant is non-zero, i.e., A1 exists if and only if |A| 0. For example, let B be the inverse of the matrix A, then AB = BA = I or |AB| = I or |A|.|B| = 1(|I| =1).

Hence |A| 0.

4. If a square matrix A has an inverse, then it is unique. It can also be proven by assuming there are two inverses B and C of A. We then have AB = BA = I and AC = CA = I (ii) (i)

128 WAWASAN OPEN UNIVERSITY


BBM 511 Quantitative Techniques

Pre-multiplying (i) by C, we get CAB = CI or IB = CI or B = C(as CA = I). This implies that the inverse of a square matrix is unique.

Singular matrix
A matrix is said to be singular if its determinant is equal to zero; otherwise it is non-singular. Properties of the inverse 1. The inverse of the inverse is the original matrix, i.e., (A1) = A. 2. The inverse of the transpose of a matrix is the transpose of its inverse, i.e., (At)1 = (A1)t. 3. The identity matrix is its own inverse, i.e., I1 = I. 4. The inverse of the product of two non-singular matrices is equal to the product of two inverses in the reverse order, i.e., (AB)1 = B1. A1.
-1

Method of finding inverse of a matrix The procedure of finding inverse of a square matrix A = [aij] of order n can be summarised in the following steps: 1. Construct the matrix of co-factors of each element aij in |A| as follows:

C11 C21 : : Cm1

C12 C22 : : Cm2

C1n C2n : : Cmn

In this case cofactors are the elements of the matrix.

UNIT 1 129
Basic mathematics for management

2. Take the transpose of the matrix of cofactors constructed in step 1. It is called adjoint of A and is denoted by Adj. A.

3. Find the value of |A|.

4. Apply the following formula to calculate the inverse of A. A1 = Adj A , where |A| 0. |A|

Example 6 Find the inverse of the matrix: 1 A = 2 1

[ ]
3 3 1 0 3 4

Solution: The determinant of matrix A is expanded with respect to the elements of the first row: 1 |A| = 2 1

| |
3 3 1

0 3 3 = I 1 4

| | |

3 2 3 4 1

3 2 +0 4 1

| | |
3 1

= 9 3(11) = 42.

Since |A| 0, therefore the inverse of A exists. The matrix of cofactor of elements A is: C11 = (1)1+1 M11 = 1

C12 = (1)1+2 M12 = 1

C13 = (1)1+3 M13 = 1

C21 = (1)2+1 M21 = 1

C22 = (1)2+2 M22 = 1

| | | | | | | | | |
3 1 2 1 2 1 3 1

3 = 9. 4 3 = 11. 4

3 = 5. 1

0 = 12. 4

1 0 = 4. 1 4

130 WAWASAN OPEN UNIVERSITY


BBM 511 Quantitative Techniques

C23 = (1)2+3 M23 = 1 C31 = (1)3+1 M31 = 1

C32 = (1)3+2 M32 = 1 C33 = (1)3+3 M33 = 1

| | | | | | | |
1 1 3 3 1 2

3 = 2. 1

0 = 9. 3

1 0 = 3. 2 3 3 = 9. 3

The matrix of cofactor of A is

[
Hence

C11 C12 C13 9 11 C21 C22 C23 = 12 4 C31 C32 C33 9 3

][ [

5 2 . 9

The adj. A is now constructed by taking the transpose of the cofactor matrix: 9 Adj. A = (Co-factor A)t 11 5 12 4 2 9 3 9

A1 =

Adj. A |A|

9 1 = 11 42 5

12 4 2

9 3 9

Activity G
For the matrix 1 A = 1 0

[ ]
4 2 0 0 0 2

a) Calculate A1. b) Verify (At)1 = (A1)t. c) Verify (Adj. A)1 = Adj. (A1).

UNIT 1 131
Basic mathematics for management

4.7 Solution of linear simultaneous equations


As mentioned earlier in the previous section, matrix algebra is useful in solving a set of linear simultaneous equations involving more than two variables. Now the procedure for getting the solution will be demonstrated. Consider the set of linear simultaneous equations 2x + 5y 2z = 3; x + y z = 5 and 3x 2y + z = 10.

These equations can also be solved by using ordinary algebra. However, to demonstrate the use of matrix algebra, the first step is to write the given system of equations in matrix form as follows: AX = B where A is known as the coefficient matrix in which coefficients of x are written in the first column, coefficients of y in the second column and the coefficients of z in the third column. X is the matrix of unknown variables x, y and z, and B is the matrix formed with the right hand terms in equations which do not involve unknowns x, y and z.

Generalising the situation, let us consider m linear equations in n unknowns x1, x2, , xn, a11 x1 + a12 x2 + a1n xn = b1 a21 x1 + a22 x2 + a2n xn = b2 am1 x1 + am2 x2 + amn xn = bm

Writing this system of equations in matrix form, AX = B where a11 a12 a1n a a22 a2n A = 21 am1 am2 amn

mn

132 WAWASAN OPEN UNIVERSITY


BBM 511 Quantitative Techniques

x1 x2 X= : : xn

[]

B=

[]
b1 b2 : : : bm

n1

m1

Classification of linear equations


If matrix B is a zero matrix, i.e., B = 0, then the system AX = 0 is said to be a homogeneous system. Otherwise, the system is said to be non-homogeneous.

Homogeneous linear equations


When the system is homogeneous, i.e., b1 = b2 = = bm = 0, the only possible solution is X = 0 or x1 = x2 =..xn = 0. It is called a trivial solution. Any other solution, if it exists, is called a non-trivial solution of the homogeneous linear equations. In order to solve the equation AX = 0, we perform an elementary operation or transformation on the given coefficient matrix A which does not change the order of the matrix. An elementary operation is any one of the following three types: 1. The interchange of any two rows (or columns). 2. The multiplication (or division) of the elements of any row (or column) by any non-zero number, e.g., the Ri (row i) can be replaced by KRi(K 0). 3. The addition of the elements of any row (or column) to the corresponding elements of any other row (or column) multiplied by any number, e.g., R i (row i) can be replaced by Ri + KR j where R j is the row j and K 0.

The elementary operation is called row operation if it applies to rows and column operation if it applies to columns.

UNIT 1 133
Basic mathematics for management

For the purpose of applying these elementary operations, we form another matrix called augmented matrix as shown below: a11 a12 a1n . b1 a21 a22 a2n . b2 [A:B] = am1 am2 amn . bm

Solution method We shall apply Gauss-Jordon method (also called Triangular Form Reduction method) to solve homogeneous linear equations. In this method, the given system of linear equations is reduced to an equivalent simpler system (i.e., system having the same solution as the given one). The new system looks like: x1 + b1x2 + c1x3 = d1 x 2 + c 2x 3 = d 2 x 3 = d 3. This method helps, not only to find solution to homogeneous equations but also to non-homogeneous system of equations having any number of unknowns.

Example 7 Solve the following system of equations using Gauss Jordon method. x1 + 3x2 x3 = 0 2x1 x2 + 4x3 = 0 x1 11x2 + 14x3 = 0

Solution: The given system of equations in matrix form is:

1 2 1

3 1 11

2 4 14

][ ] [ ]

x1 0 x2 = 0 or AX = 0. x3 0

The augmented matrix becomes 1 3 [A:O] = 2 1 1 11

2 : 0 4 :0 . 14 : 0

134 WAWASAN OPEN UNIVERSITY


BBM 511 Quantitative Techniques

Applying elementary row operations, R2 R2 2R1 R3 R3 R1

The new equivalent matrix is:

[ [

1 0 0

3 2 : 0 7 8 :0 . 14 16 : 0

Again applying R3 R3 2R2, the new equivalent matrix is: 1 0 0 3 7 0 2 : 0 8 :0 . 0 :0

The equations equivalent to the given system of equations obtained by elementary row operations are: x1 + 3x2 2x3 = 0 8 7x2 + 8x3 = 0 or x2 x3 = 0. 7

The last equation, though true, is redundant and the system is equivalent to x1 + 3x2 2x3 = 0 8 x2 x3 = 0 7

This is not in triangular form because the number of equations is less than the number of unknowns. This system can be solved in terms of x3 by assigning an arbitrary constant value, k to it. The general solution to the given system is given by Let x3 = k, k (any real number), Then x2 = 8k 7

x1 = 2k 3

()

8k 10k = 7 7

UNIT 1 135
Basic mathematics for management

Activity H Solve the following system of equations using Gauss-Jordon method. 1. 4x1 + x2 = 0 8x1 + 2x2 = 0 x1 2x2 + 3x3 = 0 2x1 + 5x2 + 6x3 = 0

2.

Non-homogeneous linear equations


The non-homogeneous linear equations can be solved by any of the following three methods: 1. Matrix Inverse method. 2. Cramers method. 3. Gauss-Jordon method.

Again, for the purpose of demonstrating the above solution methods, we shall consider three equations with three unknowns.

1. Matrix inverse method


Let AX = B be the given system of linear equations, and A1 be the inverse of A. Pre-multiplying both sides of the equation by A1, A1(AX) = A1B (A1A)X = A1B IX = A1B X = A1B where I is the identity matrix. The value of X gives the general solution to the given set of simultaneous equations. This solution is thus obtained by (a) first finding A1, and (b) post multiplying A1 by B. When the system has a solution, it is said to be consistent, otherwise it is inconsistent. A consistent system has either just one solution or infinitely many solutions.

136 WAWASAN OPEN UNIVERSITY


BBM 511 Quantitative Techniques

Example 8 The daily cost, C of operating a hospital, is a linear function of the number of in-patients, I and out-patients, P, plus a fixed cost a, i.e., C = a + b P + dI. Given the following data for three days, find the values of a, b and d by setting up a linear system of equations and using the matrix inverse. Day (in RM) 1 2 3 Cost 6,950 6,725 7,100 No. of in-patients, I 40 35 40 No. of out-patients, P 10 9 12

Solution: Based on the given daily cost equation, the system of equations for the cost of three days can be written as: a + 10b + 40d = 6,950 a + 9b + 35d = 6,725 a + 12b + 40d = 7,100 This system can be written in the matrix form as follows:

1 10 40 1 9 35 1 12 40

][ ] [ ]
a 6,950 b = 6,725 c 7,100

which is of the form AX = B, where 1 10 40 a 6,950 A= 1 9 35 ; X = b and B 6,725 . 1 12 40 c 7,100

] [] [ ] ][
5 0 5

The inverse of a matrix A is obtained as follows: C11 C12 C13 t 60 Adj. A = C21 C22 C23 = 80 C31 C32 C33 10

3 t 60 80 10 2 = 5 0 5 1 3 2 1

][

UNIT 1 137
Basic mathematics for management

1 |A| = 1 1

10 9 12

40 9 35 = 1 12 40

35 1 10 40 1

| |

35 1 + 40 40 1

| |

9 12

= (360 420) 10(40 35) + 40(12 9) = 10 0.

Since |A| 0, therefore inverse of matrix A exists and is computed as Adj. A |A| 60 1 5 10 3

A1 =

80 0 2

10 5 1

]
10 5 1 6,950 6,725 7,100

X = A1B

or

[ ] [ ][ ] [ ] [ ][ ]
a 60 1 b = 5 10 3 c 1 10 1 10 80 0 2 = 60 6,950 80 6,725 + 10 7,100 5 6,950 + 0 6,725 5 7,100 3 6,950 + 2 6,725 + 1 7,100 50,000 5000 750 = 75 300 30 =

or a = 5000, b = 75 and d = 30.

Activity I
A salesman has the following record of sales during three months for three items A, B and C, which have different rates of commission. Sales of units A 90 150 60 B 100 50 100 C 20 40 30 Total commision drawn (in RM) 800 900 850

Months January February March

138 WAWASAN OPEN UNIVERSITY


BBM 511 Quantitative Techniques

Find out the rates of commission on items A,B and C.

2. Cramers method
When the number of equations is equal to the number of unknowns and the determinant of the coefficients has non-zero value, then the system has a unique solution which can be found by using Cramers formula. xj = Dj , j = 1, 2, , n D

where D = |aij| and determinant Dj is obtained from D by replacing column j by the column of constant terms (i.e., matrix B).

Example 9 An automobile company uses three types of steel, S1, S2 and S3 for producing three different types of cars C1, C2 and C3. Steel requirements (in tons) for each type of car and the total available steel of all the three types is summarised in the following table. Types of car C1 2 1 3 C2 3 1 2 C3 4 2 1

Types of steel S1 S2 S3

Total steel available 29 13 16

Determine the number of cars of each type which can be produced.

Solution: Let x1, x2 and x3 be the number of cars of the types C1, C2 and C3 respectively which can be produced. The system of three linear equations is: 2x1 + 3x2 + 4x3 = 29 x1 + x2 + 2x3 = 13 3x1 + 2x2 + x3 = 16. These equations can also be represented in matrix form as shown below:

[ ][ ] [ ]
2 3 4 1 1 2 3 2 1

x1 29 x2 = 13 . x3 16

UNIT 1 139
Basic mathematics for management

The determinant of the coefficients matrix is 2 3 4 1 D= 1 1 2 =2 2 3 2 1

| | | | |

| | | | | |
2 1 3 1 3 2 1 +4 1 3 1 2

= 2(1 4) 3(1 6) + 4(2 3) = 5 0.

Applying Cramers method, 29 3 D1 1 = 13 1 D 5 16 2 2 29 D2 1 = 1 13 D 5 3 16 4 2 =2 1 4 2 =3 1

x1 =

x2 =

x3 =

2 3 29 D3 1 = 1 1 13 = 4. D 5 3 2 16

| | |

Hence, the number of cars of types C1, C2 and C3 which can be produced are 2, 3 and 4 respectively.

Activity J
A firm makes two products A and B. Each product requires production time in each of the two departments I and II as shown below: Time taken (in hrs/week) Dept I 5 6 Dept II 4 2

Product A B

Total time available is 80 hours and 60 hours in departments I and II respectively. Determine the number of units of products A and B which should be produced.

140 WAWASAN OPEN UNIVERSITY


BBM 511 Quantitative Techniques

4.8 Applications of matrices


Markov models
A particular mathematical model which is concerned with the brand-switching behaviour of consumers who are essentially repeat buyers of the product, is known as Markov brand-switching model. These models help in predicting the market share of a product at time period t, if the market share at the time period (t1) is known. Markov models have also been used in the study of (1) equipment maintenance and failure probability (2) stock market price movements, etc. The general expression for forecasting the buying levels at time t = n + 1 is given by Rn+1 = Rnp P11 P12 P1n P21 P22 P2n where p = Pm1 Pm2 Pmn

]
n j

is the matrix of transition probabilities. Each element of it represents the probability that a customer will change his liking from one brand to another in his next purchase. This is the reason for calling them transition probabilities and Pij = 1, R = matrix of order (1 n) representing the buying levels (or state probabilities) at a particular time period. If we know the buying levels at time t = 0, then we can find them at any time by solving the above equation by the relation: R1 = R0P, n = 0 R2 = R1P = (R0P) P = R0P2; n = 1 Now Rn = R0Pn; n = n 1.

Now as the time passes, i.e., n the purchasing levels (or market shares) tend to settle down to an equilibrium (or steady state). That is, once an equilibrium state is reached, there will be no change in the future market shares. Thus
n

Lt. Rn+1 = nLt. R .P n

or R = RP.

This relationship can be used to determine market shares in the long run.

UNIT 1 141
Basic mathematics for management

Example 10 Consider the following matrix of transition probabilities of a product available in the market in two brands: Brand A Brand A Brand B 0.9 0.3 Brand B 0.1 0.7

Determine the market shares of each of the brand in equilibrium position.

Solution: If the row vector (matrix having only one row) represents the market share of the two brands at equilibrium, then R = RP i.e., (r1 r2) = (r1 r2)

0.9 0.3

0.1 0.7

]
(1) (2)

r1

= 0.9r1 + 0.3r2 r2 = 0.1r1 + 0.7r2

or or

0.1r1 + 0.3r2 = 0 0.1r1 0.3r2 = 0

These are two linear homogeneous simultaneous equations. But these are not independent since one can be derived from the other. Hence, in order to solve, one more equation is needed, which is r1 + r2 = 1 (3)

This is because the market shares have been expressed in percentage, so the sum of market shares will be 1. Solving equations (1) and (2) with the help of equation (3), to get market shares in an equilibrium condition, r1 = 0.75 and r2 = 0.25.

Hence the expected market shares in an equilibrium condition for brand A will be 0.75 and that of brand B will be 0.25.

142 WAWASAN OPEN UNIVERSITY


BBM 511 Quantitative Techniques

Activity K
The purchase patterns of two brands of toothpaste can be expressed as a Markov process with the following transition probabilities: Formula A Formula A Formula B 0.90 0.05 Formula B 0.10 0.95

What are the projected market shares for the two formulas?

Input-output analysis
The method of input-output analysis was first proposed by Wassily W. Leotief in the 1930s. This method is based on the concept of economic inter-dependence, which means that every sector (or industry) of the economy is related to every other sector. That is, they are all inter-dependent and inter-related. This means, any change in one sector (such as strike) will affect all other industries to a varying degree. However, this technique does not explain or establish why such effects occur. The input-output model is based on the following assumptions: 1. An economy is decomposed into n sectors (or industries), and each of these produces only one kind of product. Each of the sectors uses the output of the other sectors as input. Let xj (j =1, 2, ..., n) be the gross production (output) of the jth sector. 2. Let aij represents RM value of the output from sector i which sector j must consume to produce one RM worth of its own product. It can be calculated as follows: RM value of the product of sector i required by sector j RM value of the total output of sector j

aij =

The aijs for all i and j can be represented in matrix form as shown below: a11 a12 a1n a a22 a2n A = 21 an1 an2 ann

nn

UNIT 1 143
Basic mathematics for management

The matrix A is the technical input-output coefficient matrix. This matrix remains unchanged so long as the structure of the economy remains unchanged. 3. There are neither shortages nor surpluses of product under consideration. In other words, the gross product of each sector is sufficient to meet the final demand as well as demands of other sectors. Let dj (j =1, 2, ..., n) be the final demand (in RM value) for the product produced by each of n sectors.

The input-output table displayed in the following table summarises information about the economy in question. Customer sectors Producers sectors Sector 1 2 i n x1 x2 xi xn Intermediate use for sectors 1 a11 a21 ai1 an1 2 a12 a22 ai2 an2 j a1j a2j aij anj n a1n a2n ain ann

Final use x1 x2 xi xn

Final demand d1 d2 di dn

If the economy is assumed to be in a state of dynamic equilibrium (i.e. neither shortages nor surpluses) so that the total output is just sufficient to meet the input needs of each sector as well as the needs of the final demand of all sectors themselves, then Output = Input = Need of each sector + Final demand xi = aijxj + di; for sector i = 1, 2, , n.
j=1 n

In matrix notation, we have X = AX + D x1 a11 a12 a1n x2 a21 a22 a2n where X ,A= and D = : xn an1 an2 ann

[] [

] []

d1 d2 . : dn

144 WAWASAN OPEN UNIVERSITY


BBM 511 Quantitative Techniques

The above equation can also be rewritten as: X = AX + D IX = AX + D IX AX = D (I A)X = D (I A)X = D X = (I A)1 D provided |I A| 0

where I is the identity matrix. The value of X gives how much each sector must produce which is just sufficient to meet the final demand as well as the demand of all sectors themselves.

Example 11 Given the following input-output table, calculate the gross output so as to meet the final demand of 200 units of Agriculture and 800 units of Industry. Input value (in RM 000) Agriculture 300 400 Industry 600 1200

Producer Agriculture Industry

Total output (in RM 000) 1000 2000

Solution: Using the notations as discussed above, a11 = RM value of the product of sector Agriculture used by Agriculture RM value of the total output of sector Agriculture 300 = 0.3. 1000

Similarly, a12 = 600 = 0.3. 2000 400 = 0.4. 1000 1200 = 0.6. 2000

a21 =

a22 =

Thus the technological matrix A and the final demand matrix D become

UNIT 1 145
Basic mathematics for management

A=

0.3 0.4

Now I A =

|I A| =

(I A)1 =

X = (I A)1D =

] [ ] [ ][ ][ [ | | [ ] [ ][ ] [ ] [ ]
0.3 200 ;D= 0.6 800 1 0 0 0.3 1 0.4 = 0.7 0.4 Adj. (I A) 1 = |I A| 0.16 0.4 0.3 0.4 0.7 200 800 1 0.4 0.3 0.16 0.4 0.7 1 320 640 0.16 2000 . 4000 = =

0.3 1 0.3 = 0.6 0 0.4 0.7 0.4

0 0.3 1 0.6 0.3 . 0.4

0.3 = 0.28 0.12 = 0.16 0 0.4

Hence, the gross output of Agriculture and Industry must be 2000 units and 4000 units respectively.

Activity L
In an economy there are two sectors A and B and the following table gives the supply and demand position of these in million RM: Consumer A 15 20 B 10 30

Producer A B

Total output 35 50

Determine the total output, if the demand is 12 for A and 18 for B.

146 WAWASAN OPEN UNIVERSITY


BBM 511 Quantitative Techniques

Summary
Matrices play an important role in quantitative analysis of managerial decisions. They also provide very convenient and compact methods of writing a system of linear simultaneous equations and solving them. These tools have also become very useful in all functional areas of management. Another distinct advantage of matrices is that once the system of equations can be set up in matrix form, they can be solved quickly using a computer. A number of basic matrix operations (such as matrix addition, subtraction, multiplication) were discussed in this section. This was followed by a discussion on matrix inversion and the procedure for finding matrix inverse. A number of examples were given in support of the above said operations and inverse of a matrix. Finally, two important applications of matrix algebra predicting market shares using Markov models and predicting the effect of a change in the output (or demand) of one sector of the economy on the output of the other sectors, using input-output models were discussed.

Keywords
Co-factor: The number Cij = (1)i+jMij is called the co-factor of element aij in A. Determinant: A unique scalar quantity associated with each square matrix. Identity matrix: A matrix in which diagonal elements are equal to 1 and all other elements are zero. Matrix: It is an array of numbers, arranged in rows and columns. Minor: The minor of an element is the determinant of the submatrix obtained from a given matrix by deleting the row and the column containing that element and is denoted by Mij. Null matrix: A matrix in which all elements are zero. Transpose matrix: A new matrix obtained by interchanging rows and columns of the original matrix.

UNIT 1 147
Basic mathematics for management

Further readings
Tan, S T (2007) Applied Mathematics for the Managerial, Life and Social Sciences, 4th edn, Belmont, CA: Thomson Brooks/Cole. Haeussler, E F, Paul R S and Wood, R J (2005) Introductory Mathematical Analysis for Business, Economics and the Life and Social Sciences, 11th edn, Upper Saddle River, NJ: Pearson Prentice Hall. Lial, M L, Hungerford, T W and Holcomb, J P (2007) Mathematics with Applications in the Management, Natural and Social Sciences, 9th edn, Boston: Pearson Addison Wesley. Raghawachari, M (1985) Mathematics for Management: An Introduction, Delhi: Tata McGraw Hill (India).

148 WAWASAN OPEN UNIVERSITY


BBM 511 Quantitative Techniques

Suggested answers to activities

Activity A
1. A) 3 3

Activity B
a)

b)

c)

[ [ [

0 2 0 2 7 1

2 1 2 1 6 4

3 7 + 4 1 3 7 4 1 3 0 5 2

][ ][ ][

6 4 6 4 2 1

3 7 = 5 3 3 7 = 5 1 3 7 = 4 1

][ ][ ][

8 5 4 3 4 3

6 9

]
0 1 0 1

] ][ ]

Activity C
2

0 2

2 3 7 +3 1 4 1

] [

6 3 0 = 4 5 4

][

4 2

6 21 18 + 8 3 12

][

9 21 = 15 7

22 15 14 23

Activity D
1. A) m p 2. C) May be a zero matrix. 3. a) The number of worker matrix = (50 20)

b) Total daily payment is (50

20)

()

30 = 1500 + 340 = 1840. 17

UNIT 1 149
Basic mathematics for management

Activity E
2 2 2 1 2 At = 1 4 , Bt = 2 4 0 2 0

[ ]
[
2 2

] ] ]

BtAt =

2 2 1 2 9 8 1 4 = 4 0 8 20 2 0 1 4 2 2 2 9 8 1 4 = 0 8 20 2 9

AB =

2 2

][ ][ ] [
]

][

(AB)t =

9 8 8 20

(AB)t = BtAt

Activity F
Given that a + b + c = 0. Thus a + b = c.

| | | | | |
a 0 b 0 b a b

b 0 a +c = a(a2) b(b2) (ab) 0 b 0 = a3 + b3 abc = (a + b)3 3a2b 3ab2 abc = c3 3ab(a + b) abc = c3 3ab(c) abc = c3 + 3abc abc = c(2ab c2)

150 WAWASAN OPEN UNIVERSITY


BBM 511 Quantitative Techniques

Activity G
a) |A| = 12 4 Adj. A = 2 0 4 1 A = 2 12 0
1

[ [

8 2 0 8 2 0

0 0 6 0 0 6

] ]

4 2 0 1 b) (A ) = 8 2 0 12 0 0 6
1

1 A = 4 0
t

1 0 2 0 0 2

|At| = 12 4 2 0 1 (A ) = 8 2 0 12 0 0 6
t 1

[ ] [
48 24 0 0 0 = 1 24 12 0

(A 1)t = (A t ) 1

c) |Adj. A| = 144.

12 1 12 (Adj. A) = 144 0
1

Adj. (A 1) =

[ ]
1 12 4 12 0 1 12 0 2 12 0 0 1 12

[ ]
1 12 4 0 12 2 12 0 1 12

0 .

(Adj. A)1 = Adj.(A1)

UNIT 1 151
Basic mathematics for management

Activity H
1.

[ |] [ | ] () [ | ] [ | ] () [ | ]
4 1 0 R2 2R1 + R2 8 2 0 4 0 0 1 1 0 0 R1 R1 0 1 0 4 0 1 0 x2 = 0; x1 = 0

4 1 0 1 4 1 0 R2 R2 R1 = (1)R2 + R1 0 4 0 4 0 1 0

2.

[ [ [

1 2

2 3 0 R2 2R1 R3 5 6 0

1 2 3 0 1 1 R2 R2 0 9 0 0 9 0 1 0 3 0 0 1 0 0 x2 = 0 x3 = k, k R

|] |] ( ) [ |]

2 3 0 R1 (2)R2 + R1 1 0 0

|]

x1 + 3x3 = 0 x1 = 3k

Activity I
Let a = rate of commission on item A b = rate of commission on item B c = rate of commission on item C

[ ][ ] [ ] [] [ ] [ ] [] [ ][ ]
90 150 60 100 50 100 20 40 30 a 800 b = 900 c 850 20 40 30
1

a 90 b = 150 c 60

100 50 100

800 900 850

a 2.49 103 b = 2.09 11.94 c

8.96 103 4.48 2.99

1.99 14.33 10.45

800 900 850

152 WAWASAN OPEN UNIVERSITY


BBM 511 Quantitative Techniques

4.39 103 = 6.48 103 15.74 103

Activity J
Let x = No. of units of product A produced y = No. of units of product B produced 5x + 6y = 80 6x + 2y = 60

[ ][ ] [ ] [ ] [ ][ ] [ ] [ ] []
5 6 6 2 x 80 = y 60 x 1 2 = y 26 6 = 6 80 5 60 1 200 26 180 = 7.69 8 6.92 7

Activity K
(r1, r2) = (r1, r2)

0.9 0.05

0.1 0.95

r1 = 0.9r1 + 0.05r2 0.1r1 0.05r2 = 0 (1) r2 = 0.1r1 + 0.95r2 0.1r1 0.05r2 = 0 (2) r1 + r2 = 1 r2 = 1 r1 (3) (3) (1) 0.1r1 0.05(1 r1) = 0 0.15r1 = 0.05 r1 = 0.33 r2 = 1 0.33 = 0.67.

UNIT 1 153
Basic mathematics for management

Activity L

A=

IA=

[ ] [ ]
15 35 10 50 20 30 35 50 ;D= 20 35 10 50 20 50 20 35 X = (I A)1D

[ ]
12 18

|I A| =

400 200 4 = 0 35(50) 35

20 10 35 50 50 (I A)1 = 4 20 20 35 35

20 10 35 50 50 = 4 20 20 35 35

[ ] [ ]

[ ]
12 18

[][ ]
A 73.5 = B 150 The total output for products A and B are 73.5 and 150 respectively.

154 WAWASAN OPEN UNIVERSITY


BBM 511 Quantitative Techniques

UNIT 1 155
Basic mathematics for management

Summary of Unit 1
This unit is developed to equip you with some basic mathematical skills to perform quantitative data analysis and generate useful information for supporting managerial decision making. You are exposed to various quantitative techniques and statistical models, functions and progressions, basic calculus as well as matrix algebra in the unit. The applications of these concepts in the business context are also emphasised throughout the whole unit.

Suggested answers to self assessment exercises

1. A sample list of steps in decision making: Observation A company makes specialised quality-testing equipment for food processing industry. e general manager has notice d that the number of late deliveries to customers seems to increase. Recognition After some time, the general manager receives a personal complaint from an important customer who su ered a second late delivery. e manager feels that the problem can no longer be ignored. Set objectives e general manager sees the problems as one of re ducing the number of late deliveries. Understanding the problem e general manager calls the manufacturing, produc tion and marketing managers to discuss the problem. e discussion is to identify the root of the problem. Determine options After identifying the cause of the problem, the next step is to consider and list out all possible options which are feasible. Evaluate options e group will evaluate each option in terms of qua ntity and quality. Select options e group will decide on the options or combination of options to be implemented. Monitor After setting the time frame, the new system is monitored and re-evaluated. If the results are not totally satisfactory, the problem is then rede ned.

2. a) Statistics consist of numerical facts that can be collected, organised, evaluated and interpreted. However, not all facts numerically stated can be analysed e ectively to generate useful information, for instance, the nominal qualitative data such as gender (0 for male and 1 for female). us, not all facts numerically stated are statis tics. b) Statistics is the science of data, not only averages. It involves variability, skewness as well as categorical data.

3. a) Frequency curves in statistics b) Motion lm Graphic model.

Descriptive model.

c) Flow chart in production control

Explanatory model. Simulation

d) Family equations describing the structure of an atom model.

4. Marketing a) Analysis of marketing research information. b) Statistical records for building and maintaining an extensive market.

Production a) Production planning, control and analysis. b) Evaluation of machine performance.

Finance, accounting and investment a) Financial forecast and budget preparation. b) Financial investment decisions.

Personal a) Labour turnover rate. b) Employment trends.

Economics a) Measurement of gross national product and input output analysis. b) Determination of business cycle, long term-term growth and seasonal uctuation.

Research and development a) Development of new product lines. b) Optimal use of resources.

Natural science a) Diagnosing the disease based on data.

b) Judging the e ciency of a particular drug for curing a certain disease.

5. Due to the complexity of problems and scenarios faced by todays managers, quantitative methods can be used as the scienti c approach towards the decision making. Quantitative tools and techniques provide means to de ne problems, compare alternative solutions and formulate hypothesis testing and prediction.

6.

e major phases of statistics are data collecti on, data organisation, presentation, analysis and interpretation. For instance, to analyse problems associated with the usage of coal to generate electricity: Data collection: Historical data to help forecast future demand and speci c data on the production of coal and electricity. Organising data: Develop a model that will forecast future demand and the potential environmental impact of various energy sources and usage. Presentation: Using the model to report on the investment in the new coal-energy system for a speci c period of time. Analysis: e assumption of the model and the solution are c arefully tested. Known data is used to make sure that the model is consistent with the current situation. is will also help to ne-tune the model and make it more accurate. Interpretation: Interpret the solutions and report the major nding. Also report on speci c ndings such as how coal shoul d be handled and processed into energy to reduce pollution and negative environmental consequences. Findings will also serve in planning future energy management.

7. a) Static models are concerned with a particular set of xed conditions and do not change in a short-term period. In a dynamic model, time plays an important role and there is an impact of change over a period of time. b) Analytical model consists of a mathematical structure and potential problems are solved using either mathematical or analytical technique. A simulation model is the experimentation on a mathematical structure of a real-life system. c) Descriptive model is a model used to describe the behaviour of a system based on data, for instance, regression models. Prescriptive models are

b) Judging the e ciency of a particular drug for curing a certain disease.

5. Due to the complexity of problems and scenarios faced by todays managers, quantitative methods can be used as the scienti c approach towards the decision making. Quantitative tools and techniques provide means to de ne problems, compare alternative solutions and formulate hypothesis testing and prediction.

6.

e major phases of statistics are data collecti on, data organisation, presentation, analysis and interpretation. For instance, to analyse problems associated with the usage of coal to generate electricity: Data collection: Historical data to help forecast future demand and speci c data on the production of coal and electricity. Organising data: Develop a model that will forecast future demand and the potential environmental impact of various energy sources and usage. Presentation: Using the model to report on the investment in the new coal-energy system for a speci c period of time. Analysis: e assumption of the model and the solution are c arefully tested. Known data is used to make sure that the model is consistent with the current situation. is will also help to ne-tune the model and make it more accurate. Interpretation: Interpret the solutions and report the major nding. Also report on speci c ndings such as how coal shoul d be handled and processed into energy to reduce pollution and negative environmental consequences. Findings will also serve in planning future energy management.

7. a) Static models are concerned with a particular set of xed conditions and do not change in a short-term period. In a dynamic model, time plays an important role and there is an impact of change over a period of time. b) Analytical model consists of a mathematical structure and potential problems are solved using either mathematical or analytical technique. A simulation model is the experimentation on a mathematical structure of a real-life system. c) Descriptive model is a model used to describe the behaviour of a system based on data, for instance, regression models. Prescriptive models are

Suggested answers to activities

Activity A
1. a) e domain x is any real number except 1 as a ny arithmetic expression cannot be divided by 0. e range y is any real n umber except 0. e domain is x = 0 given that y 0 as y 0 f or any x 0. e domain is x 4, the range is y 0 .

b) c)

2. 4p + 6q = 60 4p = 60 6q 3 p = 15 q. 2 e domain is p 0, the range is 15 3 3 q 0 q 15 q 10. 2 2

Suggested answers to activities

Activity B
1. a) 12 10 8 6 4 2 0 2 4 6 0 1 2 3 4 5 6 x y

2. b)

e graph is a quadratic graph.

C (RM 000) 35 30 ousands 25 20 15 10 5 0 0 1 2 3 4 5 6 t (years)

Suggested answers to activities

Activity C
1. f(x) = (x 4)(x + 3) f(4) = (4 4)(4 + 3) = 0. f(1) = (1 4)(1 + 3) f(4) = 5 2 = 10. f(3) = (3 4)(3 + 3) = 0.

2.

e roots of the function are given by: (x 4)(x + 3) = 0 (x 4) = 0 x=4

or or

(x + 3) = 0 x = 3.

Suggested answers to activities

Activity D
1. 2x2 8x + c = 0 a) If the equation has real roots: b2 4ac 0 (8) 4 2 c 0 64 8c 0 8c 64 c8
2

b) If the equation has equal roots: b2 4ac = 0 (8)2 4 2 c = 0 64 8c = 0 8c = 64 c=8

c) If the equation has imaginary roots: b2 4ac < 0 (8) 4 2 c < 0 64 8c < 0 8c > 64 c>8
2

2. Let x = amount of paper bought, y = amount of paper sold. a) Prot function = yp 2 + (x y)p3 xp1. b) Opportunity loss function due to overstock = (p 2 p3 )(x y).

Suggested answers to activities

Activity E
1. T15 = 12 + (15 1)2 = 12 + 48 = 40

2. a)

e total production at the end of the 15 th year is 8300. T15 = 1500 + (15 1)d 8300 = 1500 + 14d d= 8300 1500 14

= 485.7 486 sets of TV.

b)

e forecast amount of production for the 10 th year is: T10 = 1500 + (10 1)486 = 1500 + (9 486) = 5871 sets of TV.

Suggested answers to activities

Activity F
1. e common ratio, r of the GP is: r= = a) 7 49 1 7

e sum of the rst 20 terms is

s20 =

( )) 1 ( ) 49 ( ) =
49 1
1
20

1 7

18

6 7

57.17. b) e sum to innity is s = 49 1 7 49


6 7 1

57.17.

2. a) For the year 1985, population = 50. e annual compound rate is 0.01. For the year 1986, population = 50(0.01) + 50 = 50(1 + 0.01) = 50(1.01). For the year 1987, population = 50(1.01)(0.01) + 50(1.01) = 50(1.01)2. Hence for the year 2000, population = 50(1.01) 20001985 = 50(1.01)15 = 58.05 58 (in millions).

b) For the year 1985, population = 50. e annual compound rate is 0.02. For the year 1986, population = 50(0.02) + 50 = 50(1 + 0.02) = 50(1.02). For the year 1987, population = 50(1.02)(0.02) + 50(1.02) = 50(1.02)2. Hence for the year 2000, population = 50(1.02) 20001985 = 50(1.02)15 = 67.29 67 (in millions).

Suggested answers to activities

Activity A
1. a) nLt. 1+

( ) ( )

1 1 =1+ =1 n

2 n(1 n ) n2 b) nLt. = Lt. n + 1 n 1 n(1 + n )

= nLt. =1

( ) ( )
1
2 n

1+1 n

2. S = 2000 + 4000(1 e0.01x)


n

Lt. S = 2000 + 4000(1 0) = 6000

When there is in nite advertising expenditure, the sales of the product will maintain at 6000 (in RM 000).

Suggested answers to activities

Activity B
At the point x = 200 RHL = x Lt. C(x) = 15 200 = 3000 200 LHL =x Lt. C(x) = 13 200 = 2600 200 Hence RHL LHL, x = 200 is a point of discontinuity.

At the point x = 400 RHL = x Lt. C(x) = 13 400 = 5200 200 LHL = x Lt. C(x) = 10 400 = 4000 200 Since RHL LHL, x = 400 is another point of discontinuity.

Suggested answers to activities

Activity C
Let y = salary of the salesman, x is the number of items sold. e functional relationship of y and x is given by: y = 500 + 2x. e slope of the line is 2.

Suggested answers to activities

Activity D
When x = 8, y = 4(8) + 200 = 57. 8

When x = 9, y = 4(9) + 200 = 58.22. 9

Hence y 58.22 57 = = 1.22. x 91 When x = 20, y = 4(20) + 200 = 90. 20

When x = 21, y = 4(21) + 200 = 93.52. 21

Hence y 93.52 90 = = 3.52. x 21 20

e increase of stock cost will be higher as the or der size is higher.

Suggested answers to activities

Activity E
1. y = 5x dy =5 dx

2.

y = 5x2 + 3x + 2 dy = 10x + 3 dx x+1 x1


d d

3.

y=

dy (x 1) dx (x + 1) (x + 1) dx (x 1) = dx (x 1)2 = (x 1) (x + 1) (x 1)2 = 2 (x 1)2

4.

y = (x + 2)2(x2 1)4 d d dy = (x2 1)4 (x + 2)2 + (x + 2)2 (x2 1)4 dx dx dx = 2(x2 1)4(x + 2) + (x + 2)2(4)(x2 1)5(2x) = 2(x2 1)4(x + 2)8x(x + 2)2(x2 1)5 = 2(x2 1)5(x + 2)[x2 1 4x(x + 2)] = 2(x + 2) [3x2 8x 1] (x2 1)5

5.

y = (log10 x2)3 dy d = 3(log10 x2)2 (log10 x2) dx dx = 3(log10 x2)2 =

1 (2x) x 1n10
2

6 (log10 x2)2 x 1n10

6.

y = log e (3x2 5x) dy 1 d = 2 (3x2 5x) dx 3x 5x dx = 6x 5 3x2 5x

7.

y = 5x dy d = 5x log e 5 (x) dx dx = 5x log e 5

8.

y = x4elog x dy log x d 4 d log x =e (x ) + x4 (e ) dx dx dx = 4x3 elog x + x4 elog x = 4x3 elog x x3 elog x = 3x3 elog x

()
1 x

9.

y = xe

log e y = log e xe

= ex log e x 1 dy x d d = e (log e x) + log e x (ex ) y dx dx dx = ex

= ex

dy 1 = yex + log e x dx x = x e ex
x

() ( ( (

1 + ex log e x x 1 + log e x x

) ) )

1 + log e x x

Suggested answers to activities

Activity F
1. When x = 2, S = 2000e0.5(2) = 2000e1 = 735.7589. dS dS = 1000e0.5x. At x = 2, = 1000e0.5(2) = 1000e1 = 367.879. dt dt

2.

Suggested answers to activities

Activity G
p = 300 6q R=p.q = (300 3q)q = 300q 3q2 dR = 300 12q dq e slope of the revenue function = 3 12q.
When the slope of the revenue function = 0 300 12q = 0 q= 300 = 25 12

p = 300 6(25) = 150.

Suggested answers to activities

Activity H
p = 108 3 5q

q=

3 5(108 p) 3 1 = 480 160

When p = 12, q = dp 3 = 2 dq 5q =

1 5 160

()

= 15360 p dq p = q dp q 12

eq =

()
1
dp dq

( ) ()
1 160

1 15360

= 0.125.

Suggested answers to activities

Activity I
Let c = cost per hour for the train and v = speed of the train. c v2 c = kv2, where k is a constant. Given c = 75, v = 17. 75 2 v . 289 en k = 75 75 = . 2 17 289

us, c =

e revenue function per hour, r = 400v. e pro t function p = revenue cost = 400v 75 dp 150 = 400 2 v = 400 v. dv 289 289 dp =0 dv 400(289) = 770.67. 150 75 2 v . 289

( )

To maximise the pro t,

v=

Since hour.

d2p 150 = < 0, the maximum pro t is obtained when v = 770.67 km per dv2 289

Suggested answers to activities

Activity A
1. A) 3 3

Suggested answers to activities

Activity B
a)

b)

c)

[ [ [

0 2 0 2 7 1

2 1 2 1 6 4

3 7 + 4 1 3 7 4 1 3 0 5 2

][ ][ ][

6 4 6 4 2 1

3 7 = 5 3 3 7 = 5 1 3 7 = 4 1

][ ][ ][

8 5 4 3 4 3

6 9

]
0 1 0 1

Suggested answers to activities

Activity C
2

0 2

2 3 7 +3 1 4 1

] [

6 3 0 = 4 5 4

][

4 2

6 21 18 + 8 3 12

][

9 21 = 15 7

][

22 15 14 23

Suggested answers to activities

Activity D
1. A) m p 2. C) May be a zero matrix. 3. a) e number of worker matrix = (50 20)

b) Total daily payment is (50

20)

()

30 = 1500 + 340 = 1840. 17

Suggested answers to activities

Activity E
2 2 2 1 2 A = 1 4 , Bt = 2 4 0 2 0
t

[ ]
[ [
1 4

] ] ]

2 BA = 2
t t

2 2 1 2 9 8 1 4 = 4 0 8 20 2 0 2 2 2 9 8 1 4 = 0 8 20 2 9 8 20

2 AB = 2

][ ][ ] [
]

][

(AB)t =

9 8

(AB)t = BtAt

Suggested answers to activities

Activity F
Given that a + b + c = 0. a 0 b 0 b a b us a + b = c. b 0 a +c = a(a2) b(b2) (ab) 0 b 0 = a3 + b3 abc = (a + b)3 3a2b 3ab2 abc = c3 3ab(a + b) abc = c3 3ab(c) abc = c3 + 3abc abc = c(2ab c2)

| | | | | |

Suggested answers to activities

Activity G
a) |A| = 12 4 Adj. A = 2 0 4 1 2 12 0

A 1 =

[ [

8 2 0 8 2 0

0 0 6 0 0 6

] ]

b) (A 1) =

4 2 0 1 8 2 0 12 0 0 6

1 At = 4 0 |At| = 12
1

1 0 2 0 0 2

(A t) =

4 2 0 1 8 2 0 12 0 0 6

[ ]
12 1 12 144 0

(A 1)t = (A t ) 1

c) |Adj. A| = 144.

(Adj. A) 1 =

48 24 0

0 0 = 1 24 12 0

Adj. (A 1) =

(Adj. A)1 = Adj.(A1)

[ ]
1 12 4 12 0 1 12 0 2 12 0 0 1 12

[ ]
1 12 4 0 12 2 12 0 1 12

0 .

Suggested answers to activities

Activity H
1.

[ |] [ | ] () [ | ] [ | ] () [ | ]
4 1 0 R2 2R1 + R2 8 2 0 1 4 0 0 1 0 0 R1 R1 0 1 0 4 0 1 0 x2 = 0; x1 = 0

1 4 1 0 4 1 0 R2 R2 R1 = (1)R2 + R1 0 4 0 4 0 1 0

2.

[ [ [

1 2

2 3 0 R2 2R1 R3 5 6 0

1 1 2 3 0 1 R2 R2 0 9 0 0 9 0 1 0 3 0 0 1 0 0 x2 = 0 x3 = k, k R

|] |] ( ) [ |]

2 3 0 R1 (2)R2 + R1 1 0 0

|]

x1 + 3x3 = 0 x1 = 3k

Suggested answers to activities

Activity I
Let a = rate of commission on item A b = rate of commission on item B c = rate of commission on item C

[ ][ ] [ ] [] [ ] [ ] [] [ ][ ] [ ]
90 150 60 100 50 100 20 40 30 a 800 b = 900 c 850 20 40 30
1

90 a b = 150 60 c

100 50 100

800 900 850

a 2.49 103 b = 2.09 11.94 c 4.39 103 = 6.48 103 15.74 103

8.96 103 4.48 2.99

1.99 14.33 10.45

800 900 850

Suggested answers to activities

Activity J
Let x = No. of units of product A produced y = No. of units of product B produced 5x + 6y = 80 6x + 2y = 60

[ ][ ] [ ] [ ] [ ][ ] [ ] [ ] []
5 6 6 2 x 80 = y 60 x 1 2 = y 26 6 = 6 80 5 60 1 200 26 180 = 7.69 8 6.92 7

Suggested answers to activities

Activity K
(r1, r2) = (r1, r2)

0.9 0.05

0.1 0.95

r1 = 0.9r1 + 0.05r2 0.1r1 0.05r2 = 0 (1) r2 = 0.1r1 + 0.95r2 0.1r1 0.05r2 = 0 (2) r1 + r2 = 1 (3) r 2 = 1 r1 (3) (1) 0.1r1 0.05(1 r1) = 0 0.15r1 = 0.05 r1 = 0.33 r2 = 1 0.33 = 0.67.

Suggested answers to activities

Activity L

A=

IA=

[ ] [ ]
15 35 10 50 20 30 35 50 ;D= 20 35 10 50 20 50 20 35 X = (I A)1D

[ ]
12 18

|I A| =

400 200 4 = 0 35(50) 35

20 10 35 50 50 (I A)1 = 4 20 20 35 35

20 10 35 50 50 = 4 20 20 35 35

[][ ]
A 73.5 = B 150

[ ] [ ]

[ ]
12 18

e total output for products A and B are 73.5 and 150 respectively.

Self assessment exercises


1. ink of any major decision you made recently. Recall the steps taken by you to arrive at the nal decision. Prepare a list of those steps.

2. Comment on the following statements: a) Statistics are numerical statement of facts but all facts numerically stated are not statistics. b) Statistics is the science of averages.

3. What is the type of the following models? a) Frequency curves in statistics. b) Motion lms. c) Flow chart in production control. d) Family of equations describing the structure of an atom.

4. List at least two applications of statistics in each functional area of management.

5. What factors in modern society contribute to the increasing importance of quantitative approach to management?

6. Describe the major phases of statistics. Formulate a business problem and analyse it by applying these phases. 7. Explain the distinction between: a) Static and dynamic models. b) Analytical and simulation models. c) Descriptive and prescriptive models.

8. Describe the main features of the quantitative a pproach to management.

Activity A
1. Find the domain and range of each of the following functions. 1 x1

a) y =

b) y = x ; y 0 c) y = 4 x ; y 0

2. Let 4p + 6q = 60 be an equation containing variables p (price) and q (quantity). Identify the meaningful domain and range for the given function when price is considered as an independent variable.

Activity B
1. Draw the graph of the following functions: a) y = 3x 5 b) y = x2 c) y = log2 x 2. e data of machine operating cost (c) and the a ge (t) of the machine art is shown in the following table: 1 5 2 8 3 13 4 20 5 29

t (years) c (in RM 000)

a) Express operating cost as a function of the machine age. b) Sketch the graph of the function derived in (a).

Activity C
Given that f(x) = (x 4)(x + 3), then nd: 1. f(4), f(1) and f(3). 2. Roots of the function. _______________________________________________________________ _______________________________________________________________ _______________________________________________________________ _______________________________________________________________

Activity D
1. Consider the quadratic equation 2x2 8x + c = 0. What is the value of c, if the equation has a) real roots? b) equal roots? c) imaginary roots?

2. A newsboy buys papers for the price of p 1 cents per paper and sells them at a price of p2 cents per paper (p2>p1). e unsold papers at the end of the day, are bought by a wastepaper dealer for p3 price per paper(p3 < p1). a) Construct the prot function of the newsboy. b) Construct the opportunity loss function of the newsboy.

Activity E
1. Find the 15th term of an AP whose rst term is 12 and the common dierence is 2. 2. A rm produces 1500 TV sets during its rst year. If the total production of the rm at the end of the 15 th year is 8300 TV sets, then: a) Estimate the number of units if production has increased consistently in each year. b) Based on the estimate of the annual increment in production, forecast the amount of production for the 10th year.

Activity F
1. Determine the common ratio of the GP 1 1 49, 7, 1, , , . 7 49 a. Find the sum of the rst 20 terms of GP. b. Find the sum to innity of the terms of GP.

2.

e population of a country in 1985 was 50 milli on. Calculate the population in the year 2000 if the compounded annual rate of increase is (a) 1% (b) 2%.

Activity A
1. Evaluate:

a) nLt. 1+

( )
1 n

b)nLt.

( )
n2 n+1 S = 2000 + 4000{1 e(0.01)x }.

2.

e sales S (in RM 000) of a product as a func tion of advertising expenditure x is given by:

Find the limit of S as x

and interpret your result.

Activity B
e total cost of purchasing x units of an item wit hin each interval, c(x) is as follows: 15x; 0 x 200 c(x) = 13x; 200 < x 400 10x; x > 400

Find the points of discontinuity.

Activity C
Suppose a salesman is paid a xed sum of RM500 per month together with a bonus of RM2 for all items sold. Devise functional relationship for his salary and determine the slope of the line.

Activity D
Suppose, total cost, y of the stock of an item as a function of order size, x is represented by the equation 200 . x

y = 4x +

Compare the slope between x = 8 and 9 and x = 20 and 21. Interpret your results. ____________________________________________________________________ ____________________________________________________________________ ____________________________________________________________________ ____________________________________________________________________ ____________________________________________________________________ ____________________________________________________________________ ____________________________________________________________________

Activity E
Find the derivatives for the following functions: 1. y = 5x 2. y = 5x2 + 3x + 2 x+1 x1

3. y =

4. y = (x + 2)2 . (x2 1)4 5. y = (log10 x2)3 6. y = loge (3x2 5x) 7. y = 5x 8. y = x4 . elogx 9. y = xe


x

Activity F
e sales S for a product with price x is given by S = 2000e0.5x. Find: 1. Total sales revenue, S = 2000e
0.5x

at x = 2.

2. Marginal revenue,

dS at x = 2. dx

Activity G
e demand for a certain product is represented by the equation p = 300 6q where p is the price per unit and q is the number of units demanded. Find the revenue function. What is the slope of the revenue function? At what price is the marginal revenue zero?

Activity H
e demand q (in kg) for a commodity with its price p (in RM) is given by

p = 108

( )

3 . 5q

Find the elasticity of demand when the price is RM12.

Activity I e cost of fuel for running a train is proportional to the square of the speed generated in kilometres per hour, and costs RM75 per hour, at 17 kilometres per hour. What is the most economical speed, if the xed charges are RM400 per km per hour? _____________________________________________________________________ _____________________________________________________________________ _____________________________________________________________________ _____________________________________________________________________ _____________________________________________________________________

Activity A
Tick the correct alternative indicating the dimension of the matrix

[ ]
2 6 3 3 8 5 4 9 7 A) 3 3 B) 4 3

33

C) None of these.

Activity B
If matrices A and B are dened as

A=

0 2

2 3 ; 1 4

B=

7 1

6 4

3 5

then compute: a) A + B b) A B c) B A

Activity C
If two matrices A and B are dened as

A=

0 2

2 3 ; 1 4

B=

7 1

6 4

3 5

then compute 2A + 3B.

Activity D
1. If two matrices of dimension m n and n p are multiplied, then the resulting matrix is of dimension: A) m n B) n p C) m p D) None of these.

2. If A and B are two non-zero compatible matrices with respect to multiplication, then their product A) is always zero matrix. B) is never a zero matrix. C) may be a zero matrix. D) None of these.

3. A factory employs 50 skilled workers and 20 unskilled workers. e daily wages to skilled and unskilled workers are RM30 and RM17 respectively. Using matrix notation, nd: a) b) e matrix representing the number of workers. e total daily payment made to the workers.

Activity E
If two matrices A and B are dened as

A=

2 2

1 2 ; 4 0

2 2 B= 1 4 2 0

[ ]

then verify that (AB)t = Bt At.

Activity F
If a + b + c = 0, then verify the following result.

| |

a b c 0 a b = c(2ab c2) b 0 a

Activity G
For the matrix 1 A = 1 0

[ ]
4 2 0 0 0 2

a) Calculate A1. b) Verify (At)1 = (A1)t. c) Verify (Adj. A)1 = Adj. (A1).

Activity H Solve the following system of equations using Gauss-Jordon method. 1. 4x1 + x2 = 0 8x1 + 2x2 = 0 x1 2x2 + 3x3 = 0 2x1 + 5x2 + 6x3 = 0

2.

Activity I
A salesman has the following record of sales during three months for three items A, B and C, which have dierent rates of commission. Months January February March Sales of units A 90 150 60 B 100 50 100 C 20 40 30 Total commision drawn (in RM) 800 900 850

Find out the rates of commission on items A,B and C .

Activity J
A rm makes two products A and B. Each product requires production time in each of the two departments I and II as shown below: Product A B Time taken (in hrs/week) Dept I 5 6 Dept II 4 2

Total time available is 80 hours and 60 hours in departments I and II respectively. Determine the number of units of products A and B which should be produced.

Activity K
e purchase patterns of two brands of toothpaste c an be expressed as a Markov process with the following transition probabilities: Formula A Formula A Formula B 0.90 0.05 Formula B 0.10 0.95

What are the projected market shares for the two formulas?

Activity L
In an economy there are two sectors A and B and the following table gives the supply and demand position of these in million RM: Producer A B Consumer A 15 20 B 10 30 Total output 35 50

Determine the total output, if the demand is 12 for A and 18 for B.

You might also like